1
Fork 0
mirror of https://github.com/Steffo99/unisteffo.git synced 2024-10-16 06:17:32 +00:00

First commit

This commit is contained in:
Steffo 2022-02-02 05:36:02 +01:00
commit 95d78ec0a8
48 changed files with 4264 additions and 0 deletions

3
.eslintrc.json Normal file
View file

@ -0,0 +1,3 @@
{
"extends": "next/core-web-vitals"
}

1
.gitattributes vendored Normal file
View file

@ -0,0 +1 @@
*.pdf filter=lfs diff=lfs merge=lfs -text

37
.gitignore vendored Normal file
View file

@ -0,0 +1,37 @@
# See https://help.github.com/articles/ignoring-files/ for more about ignoring files.
# dependencies
/node_modules
/.pnp
.pnp.js
# testing
/coverage
# next.js
/.next/
/out/
# production
/build
# misc
.DS_Store
*.pem
# debug
npm-debug.log*
yarn-debug.log*
yarn-error.log*
# local env files
.env.local
.env.development.local
.env.test.local
.env.production.local
# vercel
.vercel
# typescript
*.tsbuildinfo

34
README.md Normal file
View file

@ -0,0 +1,34 @@
This is a [Next.js](https://nextjs.org/) project bootstrapped with [`create-next-app`](https://github.com/vercel/next.js/tree/canary/packages/create-next-app).
## Getting Started
First, run the development server:
```bash
npm run dev
# or
yarn dev
```
Open [http://localhost:3000](http://localhost:3000) with your browser to see the result.
You can start editing the page by modifying `pages/index.tsx`. The page auto-updates as you edit the file.
[API routes](https://nextjs.org/docs/api-routes/introduction) can be accessed on [http://localhost:3000/api/hello](http://localhost:3000/api/hello). This endpoint can be edited in `pages/api/hello.ts`.
The `pages/api` directory is mapped to `/api/*`. Files in this directory are treated as [API routes](https://nextjs.org/docs/api-routes/introduction) instead of React pages.
## Learn More
To learn more about Next.js, take a look at the following resources:
- [Next.js Documentation](https://nextjs.org/docs) - learn about Next.js features and API.
- [Learn Next.js](https://nextjs.org/learn) - an interactive Next.js tutorial.
You can check out [the Next.js GitHub repository](https://github.com/vercel/next.js/) - your feedback and contributions are welcome!
## Deploy on Vercel
The easiest way to deploy your Next.js app is to use the [Vercel Platform](https://vercel.com/new?utm_medium=default-template&filter=next.js&utm_source=create-next-app&utm_campaign=create-next-app-readme) from the creators of Next.js.
Check out our [Next.js deployment documentation](https://nextjs.org/docs/deployment) for more details.

5
next-env.d.ts vendored Normal file
View file

@ -0,0 +1,5 @@
/// <reference types="next" />
/// <reference types="next/image-types/global" />
// NOTE: This file should not be edited
// see https://nextjs.org/docs/basic-features/typescript for more information.

6
next.config.js Normal file
View file

@ -0,0 +1,6 @@
/** @type {import('next').NextConfig} */
const nextConfig = {
reactStrictMode: true,
}
module.exports = nextConfig

23
package.json Normal file
View file

@ -0,0 +1,23 @@
{
"name": "appuntiweb-omega",
"private": true,
"scripts": {
"dev": "next dev",
"build": "next build",
"start": "next start",
"lint": "next lint"
},
"dependencies": {
"@steffo/bluelib-react": "^4.2.0",
"next": "12.0.10",
"react": "17.0.2",
"react-dom": "17.0.2"
},
"devDependencies": {
"@types/node": "17.0.14",
"@types/react": "17.0.38",
"eslint": "8.8.0",
"eslint-config-next": "12.0.10",
"typescript": "4.5.5"
}
}

23
pages/_app.tsx Normal file
View file

@ -0,0 +1,23 @@
import "../styles/global.css"
import type { AppProps } from 'next/app'
import Link from "next/link"
import {Bluelib, Heading, LayoutThreeCol, Anchor as A} from "@steffo/bluelib-react"
function MyApp({ Component, pageProps }: AppProps) {
return (
<Bluelib theme="sophon" id="root">
<LayoutThreeCol>
<LayoutThreeCol.Center>
<Heading level={1}>
<Link href="/">
<A href="#">Unisteffo</A>
</Link>
</Heading>
<Component {...pageProps} />
</LayoutThreeCol.Center>
</LayoutThreeCol>
</Bluelib>
)
}
export default MyApp

128
pages/index.tsx Normal file
View file

@ -0,0 +1,128 @@
import { Heading, Chapter, Box, Idiomatic as I, Anchor as A, ListUnordered, ListOrdered } from '@steffo/bluelib-react'
import type { NextPage } from 'next'
import Link from 'next/link'
const Home: NextPage = () => {
return <>
<Heading level={2}>
Appunti universitari open source di Steffo
</Heading>
<Chapter>
<Box>
<Heading level={3}>
Dove sono?
</Heading>
<p>
Ciao! Benvenuto a <I>Unisteffo</I>, il sito su cui è caricato tutto il materiale per lo studio che ho utilizzato durante il mio percorso universitario!
</p>
<p>
Spero che ti possa essere utile!
</p>
</Box>
</Chapter>
<Chapter>
<Box>
<Heading level={3}>
Anno 1
</Heading>
<ListUnordered>
<ListUnordered.Item>
<Link href="/year1/analisi">
<A href="#">Analisi matematica</A>
</Link>
</ListUnordered.Item>
<ListUnordered.Item>
<Link href="/year1/algebra">
<A href="#">Algebra lineare</A>
</Link>
</ListUnordered.Item>
<ListUnordered.Item>
<Link href="/year1/programmazione1">
<A href="#">Programmazione 1</A>
</Link>
</ListUnordered.Item>
</ListUnordered>
<ListUnordered>
<ListUnordered.Item>
<Link href="/year1/architettura">
<A>Architettura dei calcolatori</A>
</Link>
</ListUnordered.Item>
<ListUnordered.Item>
<Link href="/year1/algoritmi">
<A>Algoritmi e strutture dati</A>
</Link>
</ListUnordered.Item>
</ListUnordered>
</Box>
<Box>
<Heading level={3}>
Anno 2
</Heading>
<ListUnordered>
<ListUnordered.Item>
<Link href="/year2/fisica">
<A>Fisica</A>
</Link>
</ListUnordered.Item>
<ListUnordered.Item>
<Link href="/year2/oggetti">
<A>Programmazione a oggetti</A>
</Link>
</ListUnordered.Item>
<ListUnordered.Item>
<Link href="/year2/statistica">
<A>Statistica ed elementi di probabilità</A>
</Link>
</ListUnordered.Item>
<ListUnordered.Item>
<Link href="/year2/apprendimento">
<A>Apprendimento ed evoluzione in sistemi artificiali</A>
</Link>
</ListUnordered.Item>
</ListUnordered>
<ListUnordered>
<ListUnordered.Item>
<Link href="/year2/database">
<A>Basi di dati</A>
</Link>
</ListUnordered.Item>
<ListUnordered.Item>
<Link href="/year2/calcolo">
<A>Calcolo numerico</A>
</Link>
</ListUnordered.Item>
<ListUnordered.Item>
<Link href="/year2/ottimizzazione">
<A>Ottimizzazione lineare intera</A>
</Link>
</ListUnordered.Item>
<ListUnordered.Item>
<Link href="/year2/sistemioperativi">
<A>Sistemi operativi</A>
</Link>
</ListUnordered.Item>
</ListUnordered>
</Box>
<Box>
<Heading level={3}>
Anno 3
</Heading>
<ListUnordered>
<ListUnordered.Item>
<Link href="/year3/gestione">
<A>Gestione dell'informazione</A>
</Link>
</ListUnordered.Item>
<ListUnordered.Item>
<Link href="/year3/web">
<A>Tecnologie web</A>
</Link>
</ListUnordered.Item>
</ListUnordered>
</Box>
</Chapter>
</>
}
export default Home

141
pages/year1/algebra.tsx Normal file
View file

@ -0,0 +1,141 @@
import { Heading, Chapter, Box, Idiomatic as I, Code, Anchor as A, Dialog, BringAttention as B, ListOrdered, ListUnordered} from '@steffo/bluelib-react'
import type { NextPage } from 'next'
const Algebra: NextPage = () => {
return <>
<Heading level={2}>
Analisi matematica
</Heading>
<Chapter>
<Box>
<Heading level={3}>
Introduzione
</Heading>
<p>
Ammetto di non aver seguito particolarmente il corso di <I>Algebra Lineare</I>.
</p>
<p>
Facevo molta fatica a seguire le lezioni del prof., quindi spesso facevo finta di seguire e mi mettevo a giocare a <A href="https://playhearthstone.com/" rel="nofollow">Hearthstone</A>...
</p>
<p>
Tuttavia, a casa ho sfruttato delle ottime videolezioni caricate dall'Unimore su YouTube per imparare quello che non avevo fatto in aula, con ottimi risultati: sono riuscito a passare l'esame al primo tentativo!
</p>
</Box>
</Chapter>
<Chapter>
<Box>
<Heading level={3}>
Videolezioni usate
</Heading>
<Dialog builtinColor='blue'>
<B>Gennaio 2022:</B> Come sempre, Unimore ha fatto del suo meglio per ostacolare lo studio ai suoi studenti, e quindi ha reso tutte le videolezioni su YouTube private.<br/>
<small>Quelle su Vimeo ci sono ancora, però...</small>
</Dialog>
<ListOrdered>
<ListOrdered.Item>
<A>Definizione di Spazio Vettoriale</A> (1:17:29)
</ListOrdered.Item>
<ListOrdered.Item>
<A>Sottospazi vettoriali I</A> (37:15)
</ListOrdered.Item>
<ListOrdered.Item>
<A>Sottospazi vettoriali II</A> (43:26)
</ListOrdered.Item>
<ListOrdered.Item>
<A>Sottospazi vettoriali III</A> (40:29)
</ListOrdered.Item>
<ListOrdered.Item>
<A>Lineare dipendenza e indipendenza</A> (56:12)
</ListOrdered.Item>
<ListOrdered.Item>
<A>Basi di uno spazio vettoriale I</A> (25:52)
</ListOrdered.Item>
<ListOrdered.Item>
<A>Basi di uno spazio vettoriale II</A> (48:24)
</ListOrdered.Item>
<ListOrdered.Item>
<A>Teorema di Grassmann</A> (32:36)
</ListOrdered.Item>
<ListOrdered.Item>
<A>Basi e Matrici</A> (27:06)
</ListOrdered.Item>
<ListOrdered.Item>
<A>Definizione di Applicazioni Lineari</A> (16:23)
</ListOrdered.Item>
<ListOrdered.Item>
<A>Proprietà delle Applicazioni Lineari</A> (31:58)
</ListOrdered.Item>
<ListOrdered.Item>
<A>Definizione di determinante</A> (36:43)
</ListOrdered.Item>
<ListOrdered.Item>
<A>Proprietà e metodo di triangolazione</A> (22:36)
</ListOrdered.Item>
<ListOrdered.Item>
<A>Teorema di Laplace</A> (29:03)
</ListOrdered.Item>
<ListOrdered.Item>
<A>4 applicazioni del Teorema di Laplace</A> (47:53)
</ListOrdered.Item>
<ListOrdered.Item>
<A>Spazi vettoriali euclidei reali - Parte 1</A> (28:46)
</ListOrdered.Item>
<ListOrdered.Item>
<A>Spazi vettoriali euclidei reali - Parte 2</A> (27:17)
</ListOrdered.Item>
<ListOrdered.Item>
<A>Autovalori e autovettori</A> (33:00)
</ListOrdered.Item>
<ListOrdered.Item>
<A>Polinomio caratteristico</A> (31:31)
</ListOrdered.Item>
<ListOrdered.Item>
<A>Teorema diagonalizzabilità</A> (18:49)
</ListOrdered.Item>
<ListOrdered.Item>
<A href="https://player.vimeo.com/video/291457587">Spazi affini</A> (20:46)
</ListOrdered.Item>
<ListOrdered.Item>
<A href="https://player.vimeo.com/video/291458991">Sottospazi affini</A> (21:32)
</ListOrdered.Item>
<ListOrdered.Item>
<A href="https://player.vimeo.com/video/291510181">Parallelismo e Riferimenti Affini</A> (16:57)
</ListOrdered.Item>
<ListOrdered.Item>
<A href="https://player.vimeo.com/video/291510296">Rappresentazione di Sottospazi Affini</A> (31:17)
</ListOrdered.Item>
<ListOrdered.Item>
<A href="https://player.vimeo.com/video/291510612">Spazi Euclidei</A> (35:57)
</ListOrdered.Item>
<ListOrdered.Item>
<A href="https://player.vimeo.com/video/291510964">Teoria dei ranghi</A> (9:44)
</ListOrdered.Item>
<ListOrdered.Item>
<A href="https://player.vimeo.com/video/291510862">Teoria dei ranghi 2</A> (14:44)
</ListOrdered.Item>
</ListOrdered>
</Box>
</Chapter>
<Chapter>
<Box>
<Heading level={3}>
Cheatsheet
</Heading>
<p>
Visto che all'esame era possibile utilizzare i propri appunti, ho realizzato un piccolo cheatsheet stampabile con tutti gli esercizi più comuni.
</p>
<ListUnordered>
<ListUnordered.Item>
Cheatsheet (<A href="/materials/year1/algebra/cheatsheet.tex"><Code>.tex</Code></A>, <A href="/materials/year1/algebra/cheatsheet.pdf"><Code>.pdf</Code></A>)
</ListUnordered.Item>
</ListUnordered>
<Dialog builtinColor='yellow'>
<B>Attenzione:</B> questi file non sono stati ricontrollati da quando sono stati scritti, e potrebbero contenere errori!<br/>
<small>Usali a tuo rischio e pericolo!</small>
</Dialog>
</Box>
</Chapter>
</>
}
export default Algebra

106
pages/year1/analisi.tsx Normal file
View file

@ -0,0 +1,106 @@
import { Heading, Chapter, Box, Idiomatic as I, Code, Anchor as A, Dialog, BringAttention as B, ListOrdered, ListUnordered} from '@steffo/bluelib-react'
import type { NextPage } from 'next'
const MaterialLi = ({children, file}) => {
return <ListOrdered.Item>
{children} (<A href={`/materials/year1/analisi/${file}.tex`}><Code>.tex</Code></A>, <A href={`/materials/year1/analisi/${file}.pdf`}><Code>.pdf</Code></A>)
</ListOrdered.Item>
}
const Analisi: NextPage = () => {
return <>
<Heading level={2}>
Analisi matematica
</Heading>
<Chapter>
<Box>
<Heading level={3}>
Introduzione
</Heading>
<p>
Ho adorato il corso di <I>Analisi matematica</I>.<br/>
La prof. Eleuteri è stata un'introduzione fantastica al mondo dell'università: chiara, disponibile e giusta.<br/>
Avrei voluto che tutti i corsi fossero stati così...
</p>
<p>
Durante il corso, mi sono interessato al <a href="https://it.wikipedia.org/wiki/LaTeX">LaTeX</a>, e per impararlo mi sono messo ad utilizzarlo per prendere appunti in aula.
</p>
<p>
Di conseguenza, tutte le dispense che ho realizzato sono in formato <Code>.tex</Code>, o in versione compilata in formato <Code>.pdf</Code>.
</p>
</Box>
</Chapter>
<Chapter>
<Box>
<Heading level={3}>
Materiale realizzato
</Heading>
<p>
Ho cercato di riordinare gli appunti in base a come ricordo fosse ordinato il programma. Spero di non essermi sbagliato!
</p>
<ListOrdered>
<MaterialLi file="1_sottosuccessioni">
Sottosuccessioni
</MaterialLi>
<MaterialLi file="2_successioni_per_ricorrenza">
Successioni per ricorrenza
</MaterialLi>
<MaterialLi file="3_intorni_e_asintoti">
Intorni e asintoti
</MaterialLi>
<MaterialLi file="4_limiti">
Limiti
</MaterialLi>
<MaterialLi file="5_derivate">
Derivate
</MaterialLi>
<MaterialLi file="6_punti_di_estremo">
Punti di estremo
</MaterialLi>
<MaterialLi file="7_studio_di_funzione">
Studio di funzione
</MaterialLi>
<MaterialLi file="8_serie">
Serie
</MaterialLi>
<MaterialLi file="9_numeri_complessi">
Numeri complessi
</MaterialLi>
<MaterialLi file="10_equazioni_complessi">
Equazioni di numeri complessi
</MaterialLi>
<MaterialLi file="11_primitive">
Integrali e primitive
</MaterialLi>
<MaterialLi file="12_integrazione">
Integrazione
</MaterialLi>
</ListOrdered>
<p>
Questi sono materiali extra utili per preparare l'orale.
</p>
<ListUnordered>
<MaterialLi file="X_weierstrass">
Teorema di Weierstrass
</MaterialLi>
<MaterialLi file="X_teoremiprincipali">
I 10 teoremi principali
</MaterialLi>
<ListUnordered.Item>
Successioni (<A href={`/materials/year1/analisi/X_successioni.md`}><Code>.md</Code></A>)
</ListUnordered.Item>
</ListUnordered>
<Dialog builtinColor='yellow'>
<B>Attenzione:</B> questi file non sono stati ricontrollati da quando sono stati scritti, e potrebbero contenere errori!<br/>
<small>Usali a tuo rischio e pericolo!</small>
</Dialog>
<Dialog builtinColor='yellow'>
<B>Attenzione:</B> questi file sono incompleti, e non sono quindi sufficienti per dare l'esame!<br/>
<small>Studia sul materiale, e poi usa questi per verificare quello che hai studiato.</small>
</Dialog>
</Box>
</Chapter>
</>
}
export default Analisi

View file

@ -0,0 +1,100 @@
import { Heading, Chapter, Box, Idiomatic as I, Code, Anchor as A, Dialog, BringAttention as B, ListOrdered, ListUnordered, Image } from '@steffo/bluelib-react'
import type { NextPage } from 'next'
const Programmazione1: NextPage = () => {
return <>
<Heading level={2}>
Programmazione 1
</Heading>
<Chapter>
<Box>
<Heading level={3}>
Introduzione
</Heading>
<p>
Avendo già programmato in C / C++ alle scuole superiori, non ho avuto il minimo bisogno di studiare per l'esame di Programmazione 1, e pertanto non ho preso appunti.
</p>
<p>
Il massimo che posso fare è consigliare qualche risorsa online per il C++, e allegare al sito la guida che avevo scritto al tempo per installare su Windows il compilatore C / C++ MinGW.
</p>
</Box>
</Chapter>
<Chapter>
<Box>
<Heading level={3}>
Collegamenti utili
</Heading>
<ListUnordered>
<ListUnordered.Item>
<a href="https://en.cppreference.com/w/">cppreference.com</a>
</ListUnordered.Item>
<ListUnordered.Item>
<a href="https://www.cplusplus.com/reference/">cplusplus.com</a>
</ListUnordered.Item>
<ListUnordered.Item>
<a href="https://stackoverflow.com/questions">stackoverflow.com</a>
</ListUnordered.Item>
</ListUnordered>
</Box>
</Chapter>
<Chapter>
<Box>
<Heading level={3}>
Come installare correttamente MinGW su Windows
</Heading>
<p>
Scaricate <A href="https://osdn.net/projects/mingw/downloads/68260/mingw-get-setup.exe/">l'installer ufficiale</A>, ed eseguitelo.
</p>
<Image limit="quarter" src="https://i.imgur.com/mDZSqjV.png"/>
<p>
Dovrebbe comparire questa schermata. Cliccate su <I>Install</I>, poi scegliete una cartella di installazione (ricordatevela!) e poi <I>Continue</I>. Lasciate stare le altre opzioni, dovrebbero essere tutte spuntate, tranne <I>For all users</I>, che dovrebbe essere disattivato.
</p>
<Image limit="quarter" src="https://i.imgur.com/brdw8Xy.png"/>
<p>
Aspettate che finisca il download. Pochi secondi dopo, dovrebbe finire e dovrebbe apparire un tasto <I>Continue</I>. Premetelo.
</p>
<Image limit="quarter" src="https://i.imgur.com/aPTwrxz.png"/>
<p>
Dovrebbe apparirvi questa finestra. L'installer di MinGW è una specie di gestore pacchetti (tipo <Code>apt</Code> su Ubuntu); potete quali pacchetti installare, e quindi quali funzionalità.
</p>
<Image limit="quarter" src="https://i.imgur.com/5QLSkFN.png"/>
<p>
Nel nostro caso, dovrebbero servirci <Code>mingw32-base-bin</Code> (per il C e alcune librerie C++) e <Code>mingw32-gcc-g++-bin</Code> (per il C++). Cliccate, quindi, sui due quadratini corrispondenti, e premete <I>Mark for Installation</I>. Dovrebbe comparire una freccia gialla sul quadratino.
</p>
<Image limit="quarter" src="https://i.imgur.com/zP74nks.png"/>
<p>
Ora, è il momento di installare i pacchetti. Aprite il menù <I>Installation</I>, poi premete <I>Apply Changes</I>, e di nuovo <I>Apply</I>.
</p>
<Image limit="quarter" src="https://i.imgur.com/jp4uz5B.png"/>
<p>
Lasciate che scarichi, ci vorrà un po'. Guardatevi un video nel frattempo, fatevi una partitina a qualcosa, tornate dopo circa 10 minuti.
</p>
<Image limit="quarter" src="https://i.imgur.com/Lq9IepY.png"/>
<p>
Una volta installato, dobbiamo aggiungere <Code>g++</Code> ai programmi eseguibili da Prompt dei Comandi: premete il tasto <kbd>Windows</kbd>, e scrivete <Code>PATH</Code>. Windows dovrebbe trovarvi automaticamente quell'opzione.
</p>
<Image limit="quarter" src="https://i.imgur.com/dy3b5Ub.png"/>
<p>
Dentro la finestra di <I>Proprietà del Sistema</I>, premete <I>Variabili d'ambiente</I>.
</p>
<Image limit="quarter" src="https://i.imgur.com/FjYpT1n.png"/>
<p>
Trovate la variabile d'ambiente globale <code>Path</code>, e fateci doppio click per modificarla.
</p>
<Image limit="quarter" src="https://i.imgur.com/klZQ9So.png"/>
<p>
Ora dovreste vedere l'elenco di tutte le cartelle contenenti programmi eseguibili da terminale: dobbiamo aggiungere quella di MinGW! Premete <I>Sfoglia</I>.
</p>
<Image limit="quarter" src="https://i.imgur.com/F6lBCqS.png"/>
<p>
Trovate la cartella in cui avete installato MinGW (vi avevo detto di ricordarvela!); entrateci, poi selezionate la sottocartella <Code>bin</Code> e premete <I>OK</I> su tutte le finestre che avete aperto fino ad ora, chiudendole.
</p>
<p>
Complimenti! Avete installato MinGW e potete compilare programmi C e C++ da Windows! Avete a disposizione <Code>gcc</Code> e <Code>g++</Code> sul Prompt dei Comandi, e potete finalmente creare dei file <Code>.exe</Code>!
</p>
</Box>
</Chapter>
</>
}
export default Programmazione1

BIN
public/favicon.ico Normal file

Binary file not shown.

After

Width:  |  Height:  |  Size: 25 KiB

BIN
public/materials/year1/algebra/cheatsheet.pdf (Stored with Git LFS) Normal file

Binary file not shown.

View file

@ -0,0 +1,121 @@
\documentclass{article}
\usepackage[utf8]{inputenc}
\usepackage{mathtools}
\usepackage{amssymb}
\usepackage{centernot}
\usepackage{bm}
\usepackage{fullpage}
% Iniziate a scrivere da qua in poi
\begin{document}
\section{Moltiplicazioni tra matrici}
\[
\begin{bmatrix}
a & b \\
c & d \\
\end{bmatrix}
*
\begin{bmatrix}
e & f \\
g & h \\
\end{bmatrix}
=
\begin{bmatrix}
ae + cf & be + df \\
ag + ch & bg + dh \\
\end{bmatrix}
\]
\section{Invertibilità di una matrice}
Si può verificare se una matrice \( A \) quadrata di ordine \( n \) è invertibile verificando una di queste definizioni equivalenti:
\begin{itemize}
\item Il determinante non è nullo: \( \det A\neq 0 \).
\item Il rango di \( A \) è \( n \).
\item La trasposta \( A^{T} \) è una matrice invertibile.
\item Tutte le righe/colonne di \( A \) sono linearmente indipendenti.
\item Tutte le righe/colonne di \( A \) formano una base di \( \mathbb{K} ^{n} \).
\item Il numero 0 non è un autovalore di \( A \).
\item \( A \) è trasformabile mediante algoritmo di Gauss-Jordan in una matrice con \( n \) pivot.
\end{itemize}
\section{Stabilire esistenza di funzione lineare}
Per controllare se esiste o no una funzione lineare è sufficiente verificare che sia valida la proprietà di linearità:\\
\begin{itemize}
\item Se due vettori sono linearmente indipendenti, anche i risultati della funzione devono essere linearmente indipendenti.
\end{itemize}
Può essere controllata velocemente vedendo se si verificano le seguenti condizioni:
\begin{itemize}
\item Se due vettori di ingresso sono uno multiplo dell'altro, allora anche i vettori di uscita devono essere uno multiplo dell'altro per la stessa costante.
\item Se un vettore di ingresso è dato dalla somma di (multipli di) altri, allora anche il vettore di uscita deve essere dato dalla somma di (multipli degli) stessi.
\end{itemize}
\section{Determinazione di matrice associata}
Vogliamo trovare la matrice associata (\(A\)) di una funzione rispetto a delle nuove basi, ad esempio \(< (1, 2, 3), (4, 5, 6), (7, 8, 9)\).\\
Procediamo disponendo in verticale gli elementi delle basi, in questo modo:
\[
M =
\begin{matrix}
1 & 4 & 7 \\
2 & 5 & 8 \\
3 & 6 & 9 \\
\end{matrix}
\]
Troviamo la matrice inversa con il metodo di Gauss-Jordan:
\[
...
\]
Calcoliamo il risultato di:
\[
B = M^{-1} * A * M
\]
Il risultato \(B\) sarà la nostra nuova matrice associata.
\section{Diagonalizzabilità}
Una matrice è \textsc{diagonalizzabile} se ha \textbf{tanti autovalori quanto il suo rango}.\\
Per trovare gli autovalori trovare dove il polinomio caratteristico (determinante della matrice fatta come quella qui sotto) è uguale a 0:
\[
\begin{vmatrix}
1 - x & 2 & 3 \\
4 & 5 - x & 6 \\
7 & 8 & 9 - x \\
\end{vmatrix}
= 0
\]
\section{Stabilire se una funzione è lineare}
Se tutti i termini della funzione sono \textbf{polinomi omogenei} di primo grado (non ci sono potenze superiori a 1), allora è automaticamente \textsc{lineare}.
\section{Immagine}
Le \textsc{basi dell'immagine} di una funzione sono i \textbf{vettori linearmente indipendenti} che la generano.
\section{Iniettività e suriettività}
Una funzione lineare è \textsc{iniettiva} se \textbf{il nucleo è di dimensione 0}, ovvero se l'unico valore che fa risultare 0 alla funzione è il vettore nullo.\\
\\
Una funzione lineare è \textsc{suriettiva} se la dimensione dell'immagine è minore o uguale al rango della funzione (degli input, il rango della matrice associata): \(dim(Im(F)) = rk(M_F)\).\\
\subsection{Matrici quadrate}
Se la funzione è un \textbf{automorfismo} (campo input = campo output), allora \(iniettivita' \Leftrightarrow suriettivita'\).
\section{Somma diretta}
Un sottospazio è \textsc{somma diretta} se i due sottospazi di cui viene fatta la somma \textbf{non hanno basi in comune}, e quindi \(dim(\pmb{U} \cap \pmb{W}) = 0\).
\subsection{Trovare basi che diano una somma diretta}
Per trovare basi che diano una somma diretta, è sufficiente \textbf{trovare basi linearmente indipendenti} con quelle che già abbiamo: solitamente parti della base canonica funzionano alla perfezione.
\end{document}

BIN
public/materials/year1/analisi/10_equazioni_complessi.pdf (Stored with Git LFS) Normal file

Binary file not shown.

View file

@ -0,0 +1,27 @@
\documentclass{article}
\usepackage[utf8]{inputenc}
\usepackage{mathtools}
\usepackage{amssymb}
\usepackage{centernot}
% New symbols
\let\oldsqrt\sqrt
\def\sqrt{\mathpalette\DHLhksqrt}
\def\DHLhksqrt#1#2{%
\setbox0=\hbox{$#1\oldsqrt{#2\,}$}\dimen0=\ht0
\advance\dimen0-0.2\ht0
\setbox2=\hbox{\vrule height\ht0 depth -\dimen0}
{\box0\lower0.4pt\box2}}
% End new symbols
\begin{document}
\section{Equazioni di numeri complessi}
Come possiamo fare a risolvere equazioni in numeri complessi?\\
Una possibile soluzione è quella di applicare la definizione di numero complesso \(z = a + \i b\).\\
Effettuiamo le seguenti sostituzioni:
\[Re z = a\]
\[Im z = b\]
\[z = a + \i b\]
\[zsegnato = a - \i b\]
Probabilmente giungeremo a un risultato \(= 0 + 0i\);
\end{document}

BIN
public/materials/year1/analisi/11_primitive.pdf (Stored with Git LFS) Normal file

Binary file not shown.

View file

@ -0,0 +1,129 @@
\documentclass{article}
\usepackage[utf8]{inputenc}
\usepackage{mathtools}
\usepackage{amssymb}
\usepackage{centernot}
% New symbols
\let\oldsqrt\sqrt
\def\sqrt{\mathpalette\DHLhksqrt}
\def\DHLhksqrt#1#2{
\setbox0=\hbox{$#1\oldsqrt{#2\,}$}\dimen0=\ht0
\advance\dimen0-0.2\ht0
\setbox2=\hbox{\vrule height\ht0 depth -\dimen0}
{\box0\lower0.4pt\box2}}
\newcommand{\iu}{\mathrm{i}\mkern1mu}
\DeclarePairedDelimiter\abs{\lvert}{\rvert}
\DeclarePairedDelimiter\norm{\lVert}{\rVert}
\makeatletter
\let\oldabs\abs
\def\abs{\@ifstar{\oldabs}{\oldabs*}}
\let\oldnorm\norm
\def\norm{\@ifstar{\oldnorm}{\oldnorm*}}
\makeatother
\newcommand*{\Value}{\frac{1}{2}x^2}
\newcommand{\intab}{\int_a^b}
% End new symbols
\begin{document}
\section{Proprietà dell'integrale}
Siano \(f, g : [a, b] \to \mathbb{R}\).\\
Siano \(\alpha, \beta, a, r, b \in \mathbb{R}\).\\
Allora:
\begin{itemize}
\item \(\alpha f + \beta g\) è integrabile:\\
\[\int_a^b (\alpha f(x) + \beta g(x)) dx = \alpha \int_a^b f(x) dx + \beta \int_a^b g(x) dx\]
\item Se \(a \leq r \leq b\), allora f è integrabile su \([a, r]\) e \([r, b]\), e in particolare:
\[\int_a^b f(x) dx = \int_a^r f(x) dx + \int_r^b f(x) dx\]
\item Se \(f \geq g\), allora \(\int_a^b f(x) dx \geq \int_a^b g(x) dx\).
\item Se \(f\) è integrabile in \([a, b]\), allora \(\abs{f}\) è integrabile (ma non il contrario!).
\end{itemize}
\section{Teorema della media integrale}
\subsection{Prima parte}
\paragraph{Ipotesi}
\(f\) integrabile su \([a, b]\)
\paragraph{Tesi}
\[inf f \leq \frac{1}{b - a} \int_a^b f(x) dx \leq sup f\]
\paragraph{Dimostrazione}
Sappiamo che \(inf f \leq f(x) \leq sup f\).\\
Per la 3a proprietà dell'integrale, allora:
\[\intab inf f dx \leq \intab f(x) dx \leq \intab sup f dx\]
Possiamo portare fuori le costanti per la 1a proprietà:
\[inf f \intab 1 dx \leq \intab f(x) dx \leq sup f \intab 1 dx\]
Allora:
\[inf f (b - a) \leq \intab f(x) dx \leq sup f (b - a)\]
E se \(b - a \neq 0\)...
\[inf f \leq \frac{1}{b - a} \int_a^b f(x) dx \leq sup f\]
\subsection{Seconda parte}
\paragraph{Ipotesi}
\begin{itemize}
\item \(inf f \leq \frac{1}{b - a} \int_a^b f(x) dx \leq sup f\)
\item \(f\) continua su \([a, b]\)
\end{itemize}
\paragraph{Tesi}
\(\exists z : \frac{1}{b - a} \int_a^b f(x) dx = f(z)\)
\paragraph{Dimostrazione}
Cambiamo forma alla tesi:
\[\exists z : \intab f(x) dx = f(z) * (b - a)\]
Se la funzione è continua in \([a, b]\), per il teorema di Weierstrass sappiamo che:
\[\exists x_m, x_M : min f = m = f(x_m) \leq f(x) \leq f(x_M) = M = max f\]
Per la prima ipotesi, allora:
\[min f = inf f \leq \frac{1}{b - a} \intab f(x) dx \leq sup f = max f\]
Essendoci un minimo e un massimo, ed essendo la funzione continua, possiamo dire per il teorema dei valori intermedi che:
\[\exists z : \frac{1}{b - a} \int_a^b f(x) dx = f(z)\]
\section{Funzione primitiva}
Sia \(f : [a, b] \to \mathbb{R}\).\\
Si dice che \(G\) è \textbf{primitiva} di \(f\) se:
\begin{itemize}
\item \(G\) è \textsc{derivabile}
\item \(\forall x \in [a, b] G' = f(x)\)
\end{itemize}
\subsection{Proposizione}
Due primitive della stessa funzione definite sullo stesso intervallo differiscono per una costante.
\paragraph{Dimostrazione}
\(G_1, G_2\) primitive di \(f\)
\[\forall x \in \mathbb{R}, G_1'(x) = f(x), G_2'(x) = f(x)\]
\[G_1'(x) - G_2'(x) = 0\]
\[(G_1 - G_2)'(x) = 0\]
\[G_1 = G_2 + C\]
\subsubsection{Se non si è su un intervallo...}
Esistono primitive di una funzione che non differiscono per una costante, ma per qualcosa di più.
\paragraph{Esempio}
\[G_1(x) = \begin{cases}
log(x) \qquad se\ x > 0\\
log(-x) \qquad se\ x < 0
\end{cases}\]
\[G_2(x) = \begin{cases}
1 + log(x) \qquad se\ x > 0\\
log(-x) \qquad se\ x < 0
\end{cases}\]
\subsection{Funzioni senza primitiva}
\[\delta(x)\qquad delta\ di\ Dirac\]
\paragraph{Dimostrazione}
Per assurdo, immaginiamo esista una primitiva \(F\) di \(f\).\\
Negli intervalli \(]-\infty, 0[\) e \(]0, +\infty[\) si ha che \(F'(x) = 0\), e quindi che la funzione è costante.\\
Se la funzione è una \textsc(primitiva), significa che dev'essere \textsc{derivabile}, e quindi \textsc{continua}.\\
Ma la funzione originale non è continua, perchè ha un salto in \(x = 0\). Assurdo.
\section{Integrale indefinito}
\[\int f(x) dx\]
L'integrale indefinito qui sopra indica l'insieme di tutte le primitive di \(f(x)\).\\
\\
Esistono funzioni che hanno primitiva, ma non è esprimibile:
\[\int \frac{sin t}{t} dt\]
\end{document}

BIN
public/materials/year1/analisi/12_integrazione.pdf (Stored with Git LFS) Normal file

Binary file not shown.

View file

@ -0,0 +1,70 @@
\documentclass{article}
\usepackage[utf8]{inputenc}
\usepackage{mathtools}
\usepackage{amssymb}
\usepackage{centernot}
% New symbols
\let\oldsqrt\sqrt
\def\sqrt{\mathpalette\DHLhksqrt}
\def\DHLhksqrt#1#2{
\setbox0=\hbox{$#1\oldsqrt{#2\,}$}\dimen0=\ht0
\advance\dimen0-0.2\ht0
\setbox2=\hbox{\vrule height\ht0 depth -\dimen0}
{\box0\lower0.4pt\box2}}
\newcommand{\iu}{\mathrm{i}\mkern1mu}
\DeclarePairedDelimiter\abs{\lvert}{\rvert}
\DeclarePairedDelimiter\norm{\lVert}{\rVert}
\makeatletter
\let\oldabs\abs
\def\abs{\@ifstar{\oldabs}{\oldabs*}}
\let\oldnorm\norm
\def\norm{\@ifstar{\oldnorm}{\oldnorm*}}
\makeatother
\newcommand*{\Value}{\frac{1}{2}x^2}
\newcommand{\intab}{\int_a^b}
% End new symbols
\begin{document}
\section{\(\delta < 0, denominatore II grado\)}
\[\int \frac{1}{x^2 + 4x + 9} dx\]
Osserviamo che \(\int \frac{1}{x^2 + 1} dx = \arctan x + c\).
Provo allora a costruire qualcosa di simile all'arcotangente.
\[\int \frac{1}{x^2 + 4x + 9} dx = \int \frac{1}{x^2 + 4x + 4 + 5} dx = 5 \int \frac{1}{\frac{(x + 2)^2}{5} + 1} = 5 \arctan (\frac{x + 2}{\sqrt{5}}) + c\]
\section{Integrale generalizzato}
Vogliamo ampliare la nostra definizione di integrale, applicandolo a una \(f\) non limitata.
\[\int_a^{b-\epsilon} f(x) dx\]
Ha senso; la funzione è limitata in \([a, b - \epsilon]\).\\
Allora, possiamo fare l'integrale \textbf{generalizzato} o improprio, se \textsc{esiste} ed è \textsc{finito}:
\[\lim_{\epsilon \to 0^+} \int_a^{b - \epsilon} f(x) dx = \int_a^b f(x) dx\]
\subsection{Esercizi}
\subsubsection{Uso di parametri}
Dire per quali valori del \textit{parametro} \(\alpha\)...
\[\int_0^1 \frac{1}{x^{\alpha}} dx\]
Per \(\alpha \leq 0\), si ha che \(\int_0^1 x^-\alpha dx\), e quindi è un integrale standard.\\
Per \(\alpha > 0\), si ha che \(\int_0^1 \frac{1}{x^\alpha} dx\).\\
C'è un problema in \(x = 0\); la funzione non è limitata! Usiamo allora la definizione di integrale generalizzato.\\
\subsubsection{Calcolo integrali generalizzati con la definizione}
\textit{Calcola} l'integrale...
\[\int_0^1 \frac{1}{x^\alpha} dx = \lim_{\epsilon \to 0^+} \int_\epsilon^1 \frac{1}{x^\alpha} dx\]
Trovo l'insieme delle sue primitive:
\[\int x^{-\alpha} dx = \begin{cases}
\log \abs{x} + c \qquad \alpha = 1\\
\frac{x^{1-\alpha}{1 - \alpha} + c \qquad \alpha \neq 1
\end{cases}\]
Infine, applico il teorema fondamentale del calcolo:\\
Per \(\alpha = 1\):
\[\lim_{\epsilon \to 0^+} \int_\epsilon^1 \frac{1}{x^\alpha} dx = [\log \abs{x}]^1_\epsilon = \log 1 - \log \epsilon = - \log \epsilon\]
Per \(\alpha \neq 1\):
[mi sa fatica scriverlo ma è uguale a sopra... credo]
\subsubsection{Uso dei criteri}
\textit{Studiare} l'integrabilità...
\end{document}

BIN
public/materials/year1/analisi/1_sottosuccessioni.pdf (Stored with Git LFS) Normal file

Binary file not shown.

View file

@ -0,0 +1,63 @@
\documentclass{article}
\usepackage[utf8]{inputenc}
\usepackage{mathtools}
\usepackage{amssymb}
% New root
\let\oldsqrt\sqrt
\def\sqrt{\mathpalette\DHLhksqrt}
\def\DHLhksqrt#1#2{%
\setbox0=\hbox{$#1\oldsqrt{#2\,}$}\dimen0=\ht0
\advance\dimen0-0.2\ht0
\setbox2=\hbox{\vrule height\ht0 depth -\dimen0}%
{\box0\lower0.4pt\box2}}
% End new root
\begin{document}
\section{Sottosuccessioni}
Si dice \textbf{sottosuccessione} di una successione \(\{a_n\}_n\) la composizione \(a_n \circ K\) dove \(K : \mathbb{N} \to \mathbb{N}\) è strettamente crescente.\\
\((a_n \circ K)(n) = a_{K_n} = a_{2n}\)\\
Praticamente sono successioni il cui dominio non è \(\mathbb{N}\), ma solo una parte di esso, ed è dato da un'altra successione \(K_n\).\\\\
Se \(a_n \to l \in \mathbb{R}\), allora anche \(a_{K_n} \to l\).\\
Se \(\forall K : \mathbb{N} \to \mathbb{N}, a_{K_n} \to l\), allora \(a_n \to l\).\\
Se una successione ha \textbf{limite} \(l\), tutte le estratte hanno lo \textbf{stesso limite}.\\
Viceversa, se tutte le sottosuccessioni hanno lo \textbf{stesso limite} \(l\), allora anche la principale ha \textbf{limite} \(l\).\\
Se esistono due sottosuccessioni con \textbf{limiti diversi}, allora la successione di partenza \textbf{non ha limite}.\\\\
Posso utilizzare le sottosuccessioni per trovare il limite di una successione solo quando l'unione del dominio di queste dia come risultato \(\mathbb{N}\).\\
\section{Teorema di bisezione}
Se in un intervallo "di limite" c'è almeno un punto di accumulazione, allora anche dividendolo in due parti il punto di accumulazione rimarrà in almeno una di queste due.
\section{Punto limite}
Se \(a_n\) ha una sottosuccessione convergente a l, si dice che l è un \textbf{punto limite}.
\section{Enunciato teorema di Bolzano-Weierstrass}
Sia \(\{a_n\}_n\) una \textbf{successione limitata}.\\
Allora, esiste almeno una sottosuccessione \(a_{K_n}\) di \(a_n\) \textbf{convergente}.
\section{Dimostrazione teorema di B-W}
Siccome \(\{a_n\}\) è limitata, allora esistono \(\alpha_0, \beta_0 \in \mathbb{R}, \alpha_0 \leq a_n \leq \beta_0\).\\
Chiamiamo \(I_0 = [\alpha_0, \beta_0]\) l'intervallo tra questi due punti.\\
Prendiamo l'insieme \(A_0 = \{n : a_n \in I_0\}\) di tutti i punti all'interno di questo intervallo.\\
\(A_0\) contiene infiniti valori, essendo una successione in \(\mathbb{N}\).\\
Applichiamo il teorema di bisezione: il punto medio dell'intervallo è \(\mu_0 = \frac{\alpha_0 + \beta_0}{2}\).\\
L'intervallo ora risulta diviso in \(I_0 = [\alpha_0, \mu_0] \cup [\mu_0, \beta_0]\).\\
Per il teorema di bisezione, almeno uno tra \([\alpha_0, \mu_0]\) e \([\mu_0, \beta_0]\) è infinito.\\
Se l'infinito è \([\alpha_0, \mu_0]\), allora \(\alpha_1 = \alpha_0\) e \(\beta_1 = \mu_0\).\\
Se l'infinito è \([\mu_0, \beta_0]\), allora \(\alpha_1 = \mu_0\) e \(\beta_1 = \beta_0)\).\\
In ogni caso, \(\alpha_0 \leq \alpha_1 \leq \beta_1 \leq \beta_0\)\\
Creiamo un nuovo intervallo \(I_1 = [\alpha_1, \beta_1]\).\\\\
Ripetiamo il procedimento di bisezione con \(\alpha_1\) e \(\beta_1\): dovremmo ottenere ancora un risultato dimezzato.\\
Dopo n passi, otteniamo un intervallo \(I_n = [\alpha_n, \beta_n]\) infinitamente piccolo.\\
Dunque, \(\alpha_0 \leq \alpha_1 \leq \alpha_2 \leq \dots \leq \alpha_n \leq \beta_n \leq \dots \leq \beta_2 \leq \beta_1 \leq \beta_0\).\\\\
\(\beta_n - \alpha_n = \frac{\beta_0 - \alpha_0}{2^n}\)\\
\(A_n = \{m : a_m \in I_n\}\) è infinito.\\
Possiamo dimostrare per induzione che le precedenti tre righe sono vere \(\forall n \in \mathbb{N}\).
Dunque, \(\{\alpha_n\}\) è una successione \textbf{monotona crescente}, e ha limite \(l\), e \(\{\beta_n\}\) è una successione \textbf{monotona decrescente}, ed essa ha limite \(m\).\\\\
Sapendo per la \textsc{gerarchia degli infiniti} che \(\frac{\beta_0 - \alpha_0}{2^n}\) tende a 0, allora possiamo anche dire che \(\beta_n - \alpha_n\) tende a 0, quindi \(l = m\).\\
\(\forall n\), prendo \(K_n \in A_0, a_{K_n} \in I_n, \alpha_n \leq a_{K_n} \leq \beta_n\).\\
Per il \textsc{teorema dei carabinieri}, visto che \(\alpha_n\) e \(\beta_n\) tendono ad l, allora anche \(a_{K_n}\) tenderà ad l.\\\\
Se \(a_n\) è \textbf{limitata}, allora \(a_n\) ha un \textbf{punto limite}, ma non viceversa.
\end{document}

Binary file not shown.

View file

@ -0,0 +1,159 @@
\documentclass{article}
\usepackage[utf8]{inputenc}
\usepackage{mathtools}
\usepackage{amssymb}
% New root
\let\oldsqrt\sqrt
\def\sqrt{\mathpalette\DHLhksqrt}
\def\DHLhksqrt#1#2{%
\setbox0=\hbox{$#1\oldsqrt{#2\,}$}\dimen0=\ht0
\advance\dimen0-0.2\ht0
\setbox2=\hbox{\vrule height\ht0 depth -\dimen0}%
{\box0\lower0.4pt\box2}}
% End new root
\begin{document}
\section{Successioni per ricorrenza}
Una successione per ricorrenza è una successione definita stabilendo l'elemento di partenza \(a_0\) e l'espressione per il valore successivo \(a_{n+1}\).\\
E' sempre definita su una semiretta dei numeri naturali: non può esistere un valore per cui non è definito un elemento ma è definito il suo successivo.\\\\
\(
\begin{cases}
a_0 = \alpha\\
a_{n+1} = f(a_n)
\end{cases}
\)\\\\
\section{Successioni per ricorrenza monotone}
Una successione per ricorrenza è monotona se il suo risultato non ha mai punti critici, ovvero \(\)
\subsection{Esercizio}
\paragraph{Ipotesi}
\(\begin{cases}
a_0 = \alpha\\
a_{n+1} = \sqrt{a_n} + 100
\end{cases}\)\\\\
\paragraph{Tesi}
\(\forall n \in \mathbb{N}, a_n \geq 0\)
\paragraph{Dimostrazione}
Inizio dell'induzione:\\
Se \(\alpha \geq 0 \), allora \(\alpha \in S\)\\\\
Passo induttivo:\\
\(n \in S \implies n+1 \in S \)\\
\(a_n \geq 0 \implies a_{n+1} \geq 0\)\\
\(\sqrt{a_n} + 100 \geq 0\)\\
\(\sqrt{\alpha} \geq -100\)\\
\(\alpha \geq 0\)
\subsection{Esercizio}
\paragraph{Ipotesi}
\(\begin{cases}
a_0 = \alpha\\
a_{n+1} = \frac{a_n}{2} + 3
\end{cases}\)
\paragraph{Tesi}
Limite della ricorrenza.
\paragraph{Svolgimento}
\(a_0 = \alpha\)\\
\(a_1 = \frac{a_0}{2} + 3 = \frac{\alpha}{2} + 3\)\\
\(a_2 = \frac{\frac{\alpha}{2} + 3}{2} + 3 = \frac{\alpha}{4} + \frac{3}{2} + 3\)\\
\(a_n = \frac{\alpha}{2^n} + 3 * (1 + \frac{1}{2} + \frac{1}{4} + \dots + \frac{1}{2^{n-1}})\)\\
\(a_n = \frac{\alpha}{2^n} + 3 * \displaystyle\sum_{i=\alpha}^{n-1} \frac{1}{2^i}\)\\
Congetturo il valore della somma:\\
[svolgimento omesso]\\
\(= 2(1 - \frac{1}{2^n})\)\\
Ora posso calcolare il limite:\\
[svolgimento omesso]\\
\(= 6\)\\
\section{Un modo più veloce}
\(a_n \to l\)\\
\(a_{n+1} \to l\)\\
\(a_{n+1} = \frac{a_n}{2} + 3\)\\
\(\frac{a_n}{2} \to \frac{l}{2}\)\\
\(l = \frac{l}{2} + 3\)\\
\(l = 6\)\\\\
Ma \(a_n\) ha veramente limite? Se è \textbf{monotona}, ha sempre un limite.\\
Se non ha limite, non possiamo usare questo metodo, perchè darà come risultato \(\frac{\infty}{\infty}\), una forma di indecisione.
\subsection{Esempio}
\(\begin{cases}
a_0 = \alpha\\
a_{n+1} = \sqrt{a_n + 2}
\end{cases}\)
\paragraph{Svolgimento}
\(\begin{cases}
y = x\\
y = \sqrt{x + 2}
\end{cases}\)\\
\(\begin{cases}
y = x\\
x = \sqrt{x + 2}
\end{cases}\)\\
\(\begin{cases}
y = x\\
x^2 = x + 2
\end{cases}\)\\
\(\begin{cases}
y = x\\
x^2 - x - 2 = 0
\end{cases}\)\\
\(\begin{cases}
y = 2\\
x = 2
\end{cases}\)\\
\(l = 2\)
\subsection{Esercizio}
\(\begin{cases}
a_0 = \alpha\\
a_{n+1} = \sqrt{a_n + 2}
\end{cases}\)\\
Dimostrare per induzione che \(\alpha \geq 2 \implies \forall n, a_n \geq 2\).
\paragraph{Ipotesi}
\(S = \{n \in \mathbb{N} : a_n \geq 2\}\)
\paragraph{Tesi}
\(S = \mathbb{N}\)
\paragraph{Dimostrazione}
\subparagraph{Passo base}
\(
a_0 \geq 2\\
0 \in S
\)
\subparagraph{Passo induttivo}
\(
n \in S \implies n+1 \in S\\
a_n \geq 2 \implies a_{n+1} \geq 2\\
a_n \geq 2 \implies \sqrt{a_n + 2} \geq 2\\
\)\\
Per ipotesi, \(a_n \geq 2\), quindi \(a_n + 2 \geq 2 + 2 = 4\) e allora \(\sqrt{a_n + 2} \geq \sqrt{4} = 2\).
\subparagraph{Passo monotono}
Perchè la successione sia monotona, dobbiamo verificare che \(a_n+1 \leq a_n\).\\
\(\sqrt{a_n + 2} \leq a_n\)\\
Arrivo alla soluzione \(a_n \leq -1 \bigcup a_n \geq 2\).\\
Tengo solo \(a_n \geq 2\). Gli altri non sono numeri naturali.
\subparagraph{Passo finale}
\(a_n\) monotona \(\implies a_n \to l\).\\
Dunque, esiste un limite, finito o infinito che sia.\\
[todo]
\end{document}

BIN
public/materials/year1/analisi/3_intorni_e_asintoti.pdf (Stored with Git LFS) Normal file

Binary file not shown.

View file

@ -0,0 +1,77 @@
\documentclass{article}
\usepackage[utf8]{inputenc}
\usepackage{mathtools}
\usepackage{amssymb}
% New root
\let\oldsqrt\sqrt
\def\sqrt{\mathpalette\DHLhksqrt}
\def\DHLhksqrt#1#2{%
\setbox0=\hbox{$#1\oldsqrt{#2\,}$}\dimen0=\ht0
\advance\dimen0-0.2\ht0
\setbox2=\hbox{\vrule height\ht0 depth -\dimen0}%
{\box0\lower0.4pt\box2}}
% End new root
\begin{document}
\section{Intorno}
Si dice \textbf{intorno} di un numero \(x_0 \in \mathbb{R}\) un qualsiasi intervallo aperto del tipo:\\
\((x_0 - \delta, x_0 + \delta)\)
\section{Punti isolati}
Si dice \textbf{punto isolato} un numero \(x_0 \in \mathbb{R}\) se esiste un intorno \(U\) di \(x_0\) tale che al suo interno sia presente solo il punto stesso e nient'altro.
\(U \bigcup A = {x_0}\)
\section{Punti di accumulazione}
Si dice \textbf{punto di accumulazione} un punto tale che non possano esistere suoi intorni che includano il punto stesso come unico elemento.\\
Sono gli opposti dei punti isolati.
\(U \bigcap A \ {X_0} \neq \emptyset\).
\section{Definizione topologica di limite}
[todo]\\\\
\(\forall U_l\) intorno di l\\
\(\exists V_{x_0}\) intorno di \(x_0\)\\
\(: x \in V_{x_0}\)\\
\(x \neq x_0 \implies f(x) \in U_l\)
\section{Limite finito all'infinito}
\(\lim_{x \to +\infty} f(x) = l \in \mathbb{R}\)\\
\(\forall \epsilon > 0, \exists K > 0 : \forall x > K, l - \epsilon < f(x) < l + \epsilon\)\\\\
\(\lim_{x \to -\infty} f(x) = l \in \mathbb{R}\)\\
\(\forall \epsilon > 0, \exists K > 0 : \forall x < K, l - \epsilon < f(x) < l + \epsilon\)\\\\
\(\epsilon\) è l'ampiezza della "striscia" in cui stanno i valori di f(x), \(K\) è la "barriera" dei valori della x e \(l\) è il valore del limite, ovvero "l'altezza" a cui si trova la striscia.
\section{Limite infinito all'infinito}
\[\lim_{x \to +\infty} f(x) = +\infty \in \mathbb{R}\]
\[\forall H > 0, \exists K > 0 : \forall x > K, f(x) > H\]\\
\[\lim_{x \to -\infty} f(x) = +\infty \in \mathbb{R}\]
\[\forall H > 0, \exists K < 0 : \forall x < K, f(x) > H\] \\
\[\lim_{x \to +\infty} f(x) = -\infty \in \mathbb{R}\]
\[\forall H < 0, \exists K > 0 : \forall x > K, f(x) < H\]
\[\lim_{x \to -\infty} f(x) = -\infty \in \mathbb{R}\]
\[\forall H < 0, \exists K < 0 : \forall x < K, f(x) < H\]
\section{Asintoto obliquo}
La funzione f ha la retta \(y = mx + q\) come \textbf{asintoto obliquo} per \(x \to +\infty\) se:
\[\lim_{x \to +\infty} (f(x) - mx - q) = 0\]
f ha asintoto obliquo se e solo se:
\[\lim_{x \to +\infty} \frac{f(x)}{x} = m \neq 0 \neq \infty\]
\[\lim_{x \to +\infty} (f(x) - mx) = q \neq \infty\]
\\
Esempio:
\(f(x) = e^x + 2x + 1\) ha asintoto obliquo se \(x \to -\infty\):\\
\[\lim_{x \to -\infty} \frac{e^x + 2x + 1}{x} = 2\]
\[\lim_{x \to -\infty} (e^x + 2x + 1 - 2x) = 1\]
\section{Limite infinito al finito}
\[\lim_{x \to x_0} f(x) = +\infty\]
\[\forall H > 0, \exists \delta > 0 : \forall x \neq x_0, | x - x_0 | < \delta \implies f(x) > H\]
\[\lim_{x \to x_0} f(x) = -\infty\]
\[\forall H < 0, \exists \delta > 0 : \forall x \neq x_0, | x - x_0 | < \delta \implies f(x) < H\]
\section{Asintoto verticale}
\[\lim_{x \to 0^+} \log x = -\infty\]
\end{document}

BIN
public/materials/year1/analisi/4_limiti.pdf (Stored with Git LFS) Normal file

Binary file not shown.

View file

@ -0,0 +1,182 @@
\documentclass{article}
\usepackage[utf8]{inputenc}
\usepackage{mathtools}
\usepackage{amssymb}
% New symbols
\let\oldsqrt\sqrt
\def\sqrt{\mathpalette\DHLhksqrt}
\def\DHLhksqrt#1#2{%
\setbox0=\hbox{$#1\oldsqrt{#2\,}$}\dimen0=\ht0
\advance\dimen0-0.2\ht0
\setbox2=\hbox{\vrule height\ht0 depth -\dimen0}
{\box0\lower0.4pt\box2}}
% End new symbols
\begin{document}
\section{Definizione topologica di limite}
\[\lim_{x \to x_0} f(x) = l\]
\[\forall U_l \exists V_{x_0} : \forall x \neq x_0, (x \in V_{x_0} \implies f(x) \in U_l)\]
\section{Limite finito al finito}
\[\lim_{x \to x_0} f(x) = l \Leftrightarrow\]
\[\forall \epsilon > 0, \exists \delta > 0 : \forall x \neq x_0, |x - x_0| < \delta\]
Se un limite esiste, e in un certo punto il suo limite è uguale al valore del punto, allora \(f\) è \textbf{continua} in quel punto.
\section{Funzioni continue}
Sia \(f : \subseteq \mathbb{R} \to \mathbb{R}\), e sia \(x_0\) un \textit{punto di accumulazione} per il dominio \(D\) della funzione, appartenente al dominio della funzione.\\
f(x) è \textbf{continua} in \(x_0\) se:
\[\lim_{x \to x_0} f(x) = f(x_0)\]
Diciamo che è continua in generale se la formula superiore è vera \(\forall x \in D\).\\
La continuità è infatti un concetto locale: i valori esterni al dominio sono ignorati.
\subsection{Esempio}
\[f(x) =
\begin{cases}
1\quad se \quad x \geq 0\\
0\quad se \quad x < 0
\end{cases}\]
\[\lim_{x \to 1} f(x) = f(1) = 1\]
In 1, \(f\) è continua, perchè il suo limite esiste ed è uguale a 1.
\[\nexists \lim_{x \to 0} f(x)\]
In 0, \(f\) non è continua, perchè il suo limite non esiste.
\subsection{Esempio}
\[\lim_{x \to x_0} f(x)\]
\[f(x) =
\begin{cases}
1\quad se \quad x \neq 0\\
0\quad se \quad x = 0
\end{cases}\]
\[\lim_{x \to 0} f(x) = 1\]
In 0, \(f\) non è continua, perchè il suo limite esiste, ma è diverso da 0,
\subsection{Esempio}
\[f(x) = \frac{1}{x}\]
E' una funzione continua? Sì, perchè è continua per tutti i punti del suo dominio. 0, infatti, non è nel suo dominio.
\section{Definizione successionale di limite}
La \textit{definizione topologica di limite} è equivalente alla seguente definizione:
\[\lim_{x \to x_0} f(x) = l\]
\[\Updownarrow\]
\[\forall \{x_n\}_{n \neq 0 \in \mathbb{N}}; (x_n \to x_0) \implies f(x_n) \to l\]
\section{Funzioni asintotiche}
Si dice che due funzioni sono \textbf{asintotiche} per \(x \to x_0\) se:
\[\lim_{x \to x_0} \frac{f(x)}{g(x)} = 1\]
Dunque, si dice che \(f\) è asintotico a \(g\) in \(x_0\):
\[\sin x \sim x \qquad x \to 0\]
\section{Limiti notevoli}
\subsection{Seno di x su x}
\[\lim_{x \to 0} \frac{\sin x}{x} = 1\]
\[\sin x \sim x \qquad x \to 0\]
\subsubsection{Esempio}
\[sin(n) \not\sim n \qquad x \to +\infty\]
\[lim_{x \to +\infty} \frac{sin n}{n} = 0\]
\subsection{Tangente di x su x}
\[\lim_{x \to 0} \frac{\tan x}{x} = 1\]
\[\tan x \sim x \qquad x \to 0\]
\subsection{Arcotangente di x su x}
\[\lim_{x \to 0} \frac{\arctan x}{x} = 1\]
\[\arctan x \sim x \qquad x \to 0\]
\subsubsection{Esempio}
\[\lim_{n \to +\infty} \arctan \frac{1}{n^2} = 0\]
\[\arctan \frac{1}{n^2} \sim \frac{1}{n^2} \qquad n \to +\infty\]
\subsubsection{Esempio}
\[\lim_{x \to +\infty} \frac{\arctan n^2}{n^2} = 0\]
\(n^2 \to +\infty\), non tende a 0, quindi non possiamo applicare il limite notevole.
\subsection{Quello che fa un mezzo}
\[\lim_{x \to 0} \frac{1 - cos x}{x^2} = \frac{1}{2}\]
\[\lim_{x \to 0} \frac{1 - cos x}{\frac{1}{2} x^2} = 1\]
\[(1 - cos x) \sim (\frac{1}{2} x^2) \qquad x \to 0\]
\subsection{Naturale}
\[\lim_{x \to 0} \frac{e^x - 1}{x} = 1\]
\[(e^x - 1) \sim x \qquad x \to 0\]
\subsubsection{Esempio}
\[\lim_{n \to +\infty} \frac{e^\frac{1}{n} - 1}{\frac{1}{n}} = 1\]
L'argomento \(\frac{1}{n}\), per n che tende a più infinito, tende a 0; pertanto, possiamo applicare il limite notevole.
\subsubsection{Esempio}
\[\lim_{n \to +\infty} \frac{e^{\frac{1}{\sqrt{n}}} - 1}{\frac{1}{\sqrt{n}}} = 1\]
\subsubsection{Esempio}
\[\lim_{n \to +\infty} \frac{e^{\frac{1}{\sqrt{n}}}}{\frac{1}{\sqrt{n}}} = +\infty\]
\subsubsection{Esempio}
\[\lim_{n \to \infty} \frac{e^{\frac{1}{x}}}{\frac{1}{x}} \nexists\]
Questo limite non esiste, perchè per \(n \to +\infty\) vale \(+\infty\), mentre per \(n \to -\infty\) vale \(0\).
\subsubsection{Esempio}
\[\lim_{n \to +\infty} \frac{e^{\frac{1}{n^2}} - 1}{\frac{1}{n}} =\]
\[\lim_{n \to +\infty} (\frac{e^{\frac{1}{n^2}} - 1}{\frac{1}{n^2}} * \frac{1}{n}) = 0\]
\subsubsection{Altri esempi}
Non avevo voglia di scriverli, quindi li ho omessi.
\subsection{Risulta e}
\[\lim_{x \to +\infty} (1 + \frac{1}{x})^x\]
\subsection{Logaritmico}
\[\lim_{x \to 0} \frac{\log(1+x)}{x} = 1\]
\[\log(1+x) \sim x \qquad x \to 0\]
\subsubsection{Esempio}
\[\lim_{x \to +\infty} \frac{\log(1+x)}{x} = 0\]
\subsection{L'ultima}
\[\lim_{x \to 0^+} x \log x = 0\]
\subsubsection{Esempio}
\[\lim_{x \to 0^+} x^x = e^(x \log x) = 1\]
\[\lim_{x \to +\infty} x^x = e^(x \log x) = +\infty\]
\section{Esempi}
\subsubsection{Esempio}
\[\lim_{x \to +\infty} \frac{\sin x + 2x^2 + e^{-x}}{2x - 2x^2 + e^{-x}} = -1\]
Prevale \(2x^2\), perchè \(e^{-x}\) tende a 0.
\subsubsection{Esempio}
\[\lim_{x \to 0} \frac{\sin x + 2x^2 + e^{-x}}{2x - 2x^2 + e^{-x}} = 0\]
Prevale \(e^{-x}\), perchè è l'unico che non tende a 0, tendendo invece a 1.
\subsubsection{Esempio}
\[\lim_{x \to 0} \frac{| x - \pi |}{x - \pi} = -1\]
Perchè il valore assoluto diventa \(\pi - x\), e dopo prevale \(-x\).
\subsubsection{Esempio}
\[\lim_{x \to \pi} \frac{| x - \pi |}{x - \pi} = +-1\]
Dipende da che direzione ci approcciamo a \(\pi\): per \(x \to \pi^+\), il limite vale 1, ma per \(x \to \pi^-\), il limite vale -1.
\subsubsection{Esempio}
\[\lim_{x \to \pi} \frac{\cos x + 1}{(x - \pi)^2} = [0/0]\]
\[z = x - \pi\]
\[\lim_{z \to 0} \frac{\cos (z + \pi) + 1}{(z + \pi - \pi)^2}\]
\[\lim_{z \to 0} \frac{\cos z \cos \pi - \sin z \sin \pi + 1}{z^2}\]
\[\lim_{z \to 0} \frac{-\cos z + 1}{z^2} = \frac{1}{2}\]
Applichiamo il cambio di variabile in modo di avere un limite per 0: dopo, applichiamo la formula del coseno della somma.
\subsubsection{Esempio}
\[\lim_{x \to 0} \frac{log(1+x) - sin x}{x + sin x} = [0/0]\]
\[\lim_{x \to 0} \frac{log(1+x)}{x + sin x} - \frac{sin x}{x + sin x}\]
\[\lim_{x \to 0} \frac{log(1+x)}{x (1 + \frac{sin x}{x})} - \frac{sin x}{x (1 + \frac{sin x}{x})}\]
\[\frac{1}{1 + 1} - \frac{1}{1 + 1} = 0\]
Separiamo il limite in due: è un'operazione che funziona solo se nessuno dei due nuovi limiti risulta infinito o indeterminato.
\subsubsection{Esempio}
\[\lim_{x \to 0} \frac{\cos x - 2^x}{\arctan(\log(\sin \sqrt{x} + 1))} = [0/0]\]
\[\lim_{x \to 0} \frac{\cos x - x^2 + 1 - 1}{\arctan(\log(\sin \sqrt{x} + 1))}\]
Per \(x \to 0\), \(\sin \sqrt{x} \sim \sqrt{x}\); \(\log (1 + z) \sim z\); \(\arctan z \sim z\), dunque.
\[\lim_{x \to 0} \frac{\cos x - x^2 + 1 - 1}{\sqrt{x}}\]
\[\lim_{x \to 0} \frac{\cos x - 1}{\sqrt{x} - \lim_{x \to 0}\frac{x^2} + 1 - 1}{\sqrt{x}}\]
\[\lim_{x \to 0} \frac{-\frac{1}{2}x^2}{\sqrt{x}} - \lim_{x \to 0}\frac{x^2 - 1}{\sqrt{x}}\]
\[0 - 0 = 0\]
\end{document}

BIN
public/materials/year1/analisi/5_derivate.pdf (Stored with Git LFS) Normal file

Binary file not shown.

View file

@ -0,0 +1,144 @@
\documentclass{article}
\usepackage[utf8]{inputenc}
\usepackage{mathtools}
\usepackage{amssymb}
\usepackage{centernot}
% New symbols
\let\oldsqrt\sqrt
\def\sqrt{\mathpalette\DHLhksqrt}
\def\DHLhksqrt#1#2{%
\setbox0=\hbox{$#1\oldsqrt{#2\,}$}\dimen0=\ht0
\advance\dimen0-0.2\ht0
\setbox2=\hbox{\vrule height\ht0 depth -\dimen0}
{\box0\lower0.4pt\box2}}
% End new symbols
\begin{document}
\section{Definizione}
\[f : A \subseteq dom(f) \to \mathbb{R}, continua\]
\[A = [a, b]\]
f è derivabile in \(x_0\) se \textbf{esiste ed è finito} il limite del rapporto incrementale:
\[f'(x) = \lim_{h \to 0}\frac{f(x+h) - f(x)}{h}\]
f è il \textbf{coefficiente angolare} della retta tangente a \(f(x_0)\).
\subsection{Equazione retta tangente al grafico di f in \(x_0, f(x_0)\)}
\[y = f(x_0) + f'(x_0) * (x - x_0)\]
\section{Derivate particolari}
\(f = costante\); \(f' = 0\)\\
\(f = x\); \(f' = 1\)\\
\(f = x^2\); \(f' = 2x\)\\
\(f = x^n\); \(f' = nx^{n-1}\)\\
\subsubsection{Dimostrazione di \(x^n\)}
[todo]
\[\lim_{h \to 0} \frac{(x+h)^\alpha - x^\alpha}{h}\]
\[\lim_{h \to 0} x^\alpha * (\frac{\frac{(x+h)}{x}^\alpha - 1}{h})\]
\[\lim_{h \to 0} x^\alpha * (\frac{e^{\log(\frac{(x+h)}{x}^\alpha)} - 1}{h})\]
\[\lim_{h \to 0} x^\alpha * (\frac{e^{\alpha \log(\frac{(x+h)}{x})} - 1}{h})\]
\[\lim_{h \to 0} x^\alpha * (\frac{e^\frac{\alpha h}{x}) - 1}{h})\]
\subsubsection{Non derivate il valore assoluto}
Campagna pubblicitaria: chi deriva il valore assoluto muore (accademicamente).
\(|x|\) non è derivabile in \(x = 0\).
\[\lim_{h \to 0} \frac{|h|}{h} = \nexists\]
\[\lim_{h \to 0^+} \frac{|h|}{h} = 1\]
\[\lim_{h \to 0^-} \frac{|h|}{h} = -1\]
\section{Derivate sinistra e destra}
Derivata destra:
\[f_+'(x) = \lim_{h \to 0^+}\frac{f(x+h) - f(x)}{h}\]
Derivata sinistra:
\[f_-'(x) = \lim_{h \to 0^-}\frac{f(x+h) - f(x)}{h}\]
[todo: migliorare un po']
\begin{itemize}
\item Se sono uguali e finite, esiste la derivata in quel punto;\\
\item se sono diverse e almeno una delle due finita, si ha un \textbf{punto angoloso};\\
\item se sono diverse e infinite, la tangente esiste ed è completamente verticale;\\
\item se sono uguali e infinite, si forma una cuspide.
\end{itemize}
\section{Teorema di continuità}
Se \(f\) è derivabile in \(x_0\), allora \(f\) è continua.
\paragraph{Tesi}
\[\lim_{x \to x_0} f(x) = f(x_0)\]
\paragraph{Dimostrazione}
\[\lim_{x \to x_0} f(x) - f(x_0) = 0\]
\[\lim_{x \to x_0} (f(x) - f(x_0)) * \frac{x - x_0}{x - x_0}\]
\[\lim_{x \to x_0} \frac{f(x) - f(x_0)}{x - x_0} * (x - x_0)\]
\[f'(x_0) * 0 = 0\]
\subsection{Conseguenze}
\(f\) derivabile in \(x_0\) \(\implies\) \(f\) continua in \(x_0\)\\
\(f\) non continua \(\implies\) \(f\) non derivabile\\
\(f\) continua \(\centernot\implies\) \(f\) derivabile\\
\(f\) non derivabile \(\centernot\implies\) \(f\) non continua\\
\subsubsection{Esempio}
\[f(x) =
\begin{cases}
1 \qquad x > 0\\
0 \qquad x \leq 0
\end{cases}\]
Non continua in \(x = 0\), quindi non derivabile in quel punto.\\
In tutti gli altri casi, \(f'(x) = 0\).
\section{Regole di calcolo}
\[(f + g)' = f' + g'\]
\[(kf)' = kf'\]
\[(f * g)' = (f' * g) + (f * g')\]
\[(\frac{f}{g})' = \frac{(f' * g) - (f * g')}{g^2}\]
\subsection{Regola della catena}
Se \(f\) è derivabile in \(x_0\) e g è derivabile in \(f(x_0)\) e \(x_0\) è punto di accumulazione per \(dom(g \circ f)\), allora \(g \circ f\) è derivabile in \(x_0\) e vale:
\[(g \circ f)'(x_0) = g'(f(x_0)) * f'(x_0)\]
\subsubsection{Esempio}
\[f(x) = \sin^2(4 \sqrt{x} + 2\]
\[f'(x) = 2 \sin (4 \sqrt{x} + 2) * \cos (4 \sqrt{x} + 2) * (4 * \frac{1}{2 \sqrt{x}})\]
\subsubsection{Esempio}
\[f(x) = \arctan \frac{2x}{\sqrt{x^3+1}}\]
\[f'(x) = \cfrac{1}{1 + (\cfrac{2x}{\sqrt{x^3 + 1}}} * \frac{2 \sqrt{x^3 + 1} - 2x}{x^3+1} * \frac{3 x^2}{2} * \frac{1}{\sqrt{x^3+1}}\]
\subsection{Derivata della funzione inversa}
\(f : (a, b) \to \mathbb{R}\) continua e strettamente monotona \(\implies f\) invertibile\\
\(f^{-1}\) funzione di \(f\)\\
\(f\) derivabile in \(x_0 \implies f^{-1}\) derivabile in \(f(x_0) = y_0\)\\\\
\((f^{-1})'(y_0) = \frac{1}{f'(x_0)}\)\\
\subsubsection{Esempio}
\[f(x) = x + e^x\]
\[\exists f^{-1}\]
Determinare l'equazione della tangente al grafico di \(f^{-1}\) in (1, 0).\\
\[y_0 = f(x_0) = 0 + e^0 = 1\]
\[x_0 = f^{-1}(y_0) = 0\]
\[f^{-1}'(y_0) = \frac{1}{1 + e^x}\]
\[y - f^{-1}(y_0) = (f^{-1})'(y_0) * (x - y_0)\]
\[y - 0 = \frac{1}{1 + e^{1}} * (x - 1)\]
\[y = \frac{1}{1 + e} * (x - 1)\]
\section{O piccolo}
Date due funzioni \(f\) e \(g\) definite in un intorno di \(x_0\), diciamo che \(f(x) = o(g(x))\), f è \textbf{o piccolo} di \(g\) per \(x \to x_0\) se \(\lim_{x \to x_0} \frac{f(x)}{g(x)} = 0\).
\subsubsection{Esempio}
\[x^2 = o(x) \qquad x \to 0\]
Sì, perchè \(\lim_{x \to 0} \frac{x^2}{x} = 0\).
\subsubsection{Esempio}
\[sin x = o(x) \qquad x \to 0\]
No, perchè \(\lim_{x \to 0} \frac{sin x}{x} = 1\).
\subsection{Proposizione}
\[f(x) \sim g(x) \Rleftarrow f(x) = g(x) + o(g(x)) \Rleftarrow \lim_{x \to x_0} \frac{f(x)}{g(x)} = 1 \Rleftarrow \lim_{x \to x_0} - 1 = 0 \Rleftarrow \lim_{x \to x_0} \frac{f(x) - g(x)}{g(x)} = 0\]
\end{document}

BIN
public/materials/year1/analisi/6_punti_di_estremo.pdf (Stored with Git LFS) Normal file

Binary file not shown.

View file

@ -0,0 +1,97 @@
\documentclass{article}
\usepackage[utf8]{inputenc}
\usepackage{mathtools}
\usepackage{amssymb}
\usepackage{centernot}
% New symbols
\let\oldsqrt\sqrt
\def\sqrt{\mathpalette\DHLhksqrt}
\def\DHLhksqrt#1#2{%
\setbox0=\hbox{$#1\oldsqrt{#2\,}$}\dimen0=\ht0
\advance\dimen0-0.2\ht0
\setbox2=\hbox{\vrule height\ht0 depth -\dimen0}
{\box0\lower0.4pt\box2}}
\newcommand{\iu}{\mathrm{i}\mkern1mu}
% End new symbols
\begin{document}
\section{Punti di estremo}
\subsection{Massimo globale}
Si dice che \(M\) è \textbf{massimo} globale per \(f\) su \([a, b]\), e che \(x_M \in [a, b]\) è \textbf{punto di massimo} per \(f\) se:
\[\forall x \in [a, b], f(x) \leq f(x_M)\]
\subsection{Minimo globale}
Si dice che \(m\) è \textbf{minimo} globale per \(f\) su \([a, b]\), e che \(x_M \in [a, b]\) è \textbf{punto di minimo} per \(f\) se:
\[\forall x \in [a, b], f(x) \geq f(x_m)\]
\subsection{Massimo locale}
Si dice che \(M\) è \textbf{massimo locale} per \(f\) su \([a, b]\) e \(x_M\) è \textbf{punto di massimo locale} se:
\[\exists \delta > 0 : \forall x in [a, b] \cap (x_M - \delta, x_M + \delta), f(x) \leq f(x_M) = M\]
\subsection{Minimo locale}
Si dice che \(m\) è \textbf{minimo locale} per \(f\) su \([a, b]\) e \(x_M\) è \textbf{punto di minimo locale} se:
\[\exists \delta > 0 : \forall x in [a, b] \cap (x_m - \delta, x_m + \delta), f(x) \geq f(x_m) = m\]
\subsection{Massimo locale stretto}
Si dice che \(M\) è \textbf{massimo locale stretto} per \(f\) su \([a, b]\) e \(x_M\) è \textbf{punto di massimo locale stretto} se:
\[\exists \delta > 0 : \forall x in [a, b] \cap (x_M - \delta, x_M + \delta), f(x) < f(x_M) = M\]
\subsection{Minimo locale stretto}
Si dice che \(m\) è \textbf{minimo locale stretto} per \(f\) su \([a, b]\) e \(x_M\) è \textbf{punto di minimo locale stretto} se:
\[\exists \delta > 0 : \forall x in [a, b] \cap (x_m - \delta, x_m + \delta), f(x) > f(x_m) = m\]
\section{Problemi di massimo e minimo}
Dove si trovano i punti di massimo e minimo per una funzione?
\[f : [a, b] \to \mathbb{R}\]
\[f : \mathbb{R} \to \mathbb{R}\]
Si trovano dove la \textit{derivata prima si annulla}! Ma non sempre...\\
Ad esempio, \(f(x) = |x|\) ha un punto di minimo globale in \(x = 0\).\\
Inoltre, se \(f : [a, b]\), \(a\) e \(b\) sono \textit{sicuramente} punti di massimo o minimo locale, e potrebbero essere anche punti di massimo o minimo globale.
\section{Teorema di Fermat}
Sia \(f : [a, b] \to \mathbb{R}\), \textit{derivabile} in \(x_0 \in (a, b)\).\\
Se \(x_0\) è \textit{punto di estremo locale}, allora \(f'(x_0) = 0\).\\
\paragraph{Dimostrazione}
\[\exists \delta > 0 : \forall x \in (a, b) \cap (x_0 - \delta, x_0 + \delta), f(x) \geq f(x_0)\]
Se \(x < x_0\), allora \(\frac{f(x) - f(x_0)}{x - x_0} \leq 0\).\\
Se \(x > x_0\), allora \(\frac{f(x) - f(x_0)}{x - x_0} \geq 0\).\\
Passando al limite di entrambe:
\[x < x_0 \implies \lim_{x \to x_0} \frac{f(x) - f(x_0)}{x - x_0} \leq 0\]
\[x > x_0 \implies \lim_{x \to x_0} \frac{f(x) - f(x_0)}{x - x_0} \geq 0\]
Il limite appena calcolato è la derivata prima rispettivamente sinistra e destra di \(x_0\).\\
Essendo però \(f\) \textit{derivabile} in quell'intervallo, allora derivate sinistra e destra coincidono, dunque \(f'(x_0) = 0\).
\section{Teorema di Rolle}
\paragraph{Ipotesi}
Sia \(f : [a, b] \to \mathbb{R}\).\\
\(f\) \textit{continua} su \([a, b]\)\\
\(f\) \textit{derivabile} su \((a, b)\)\\
\(f(a) = f(b)\)
\paragraph{Tesi}
\[\exists c \in (a, b) : f'(c) = 0\]
\paragraph{Dimostrazione}
Essendo \(f\) \textit{continua} su \([a, b]\), essa ammette massimo e minimo per il \textsc{Teorema di Weierstrass}.
\[\exists x_m, x_M \in [a, b] : \forall x \in [a, b], f(x_m) \leq f(x) \leq f(x_M)\]
Abbiamo due casi:\\
- i due estremi coincidono con \(x_m\) e \(x_M\), creando allora una funzione costante di derivata prima sempre \(= 0\)
- altrimenti, almeno uno tra \(x_m\) e \(x_M\) è \textit{interno} all'intervallo (a, b), e per il \textsc{Teorema di Fermat} allora \(\exists c : f'(c) = 0\).
\section{Teorema di Cauchy}
\paragraph{Ipotesi}
Siano \(f, g : [a, b] \to \mathbb{R}\) tali che:
\(f, g\) \textit{continue} su \([a, b]\)\\
\(f, g\) \textit{derivabili} su \((a, b)\)
\paragraph{Tesi}
\[\exists c \in (a, b) : (f(b) - f(a)) g'(c) = (g(b) - g(a)) f'(c)\]
\paragraph{Dimostrazione}
Costruisco la funzione \(w(x) = (f(b) - f(a)) g(x) = (g(b) - g(a)) f(x)\).\\
Calcolo \(w(a)\) e \(w(b)\) (omesso per orario), e scopro \(w(a) = w(b)\).\\
Inoltre, \(w\) è \textit{continua} su \([a, b]\), e \textit{derivabile} su \((a, b)\).
Dal \textsc{Teorema di Rolle} applicato a \(w\), \(\exists c \in (a, b) : w'(c) = 0\)
\end{document}

BIN
public/materials/year1/analisi/7_studio_di_funzione.pdf (Stored with Git LFS) Normal file

Binary file not shown.

View file

@ -0,0 +1,137 @@
\documentclass{article}
\usepackage[utf8]{inputenc}
\usepackage{mathtools}
\usepackage{amssymb}
\usepackage{centernot}
% New symbols
\let\oldsqrt\sqrt
\def\sqrt{\mathpalette\DHLhksqrt}
\def\DHLhksqrt#1#2{%
\setbox0=\hbox{$#1\oldsqrt{#2\,}$}\dimen0=\ht0
\advance\dimen0-0.2\ht0
\setbox2=\hbox{\vrule height\ht0 depth -\dimen0}
{\box0\lower0.4pt\box2}}
\newcommand{\iu}{\mathrm{i}\mkern1mu}
% End new symbols
\begin{document}
\section{Studi di funzione}
\subsection{Studio di funzione classico}
\[f(x) = 2 arctan(x) - x\]
\subsubsection{Funzione \(f\)}
\paragraph{Dominio}
Il dominio in un punto è il più grande insieme possibile su cui è valida la funzione \(f\).\\\\
In questo caso, il dominio è \(\mathbb{R}\)
\paragraph{Simmetrie}
Verifichiamo se la funzione ha simmetrie: è pari? È dispari?\\\\
\(arctan(x)\) è dispari, e \(x\) è anch'esso dispari, quindi andiamo a verificare.\\
\[f(-x) = 2 arctan(-x) + x = -2 arctan(x) + x = -f(x)\]
E' dunque dispari.
\paragraph{Positività}
Troviamo dove la funzione è positiva o negativa.\\
Spesso richiede calcoli molto complessi, quindi potrebbe non valer la pena perderci tempo.\\\\
Ad esempio, in questo caso.
\paragraph{Periodicità}
Controlliamo se e dove la funzione è periodica.\\
Come per la positività, potrebbe richiedere calcoli complessi, quindi non è particolarmente importante.\\\\
Come qui.
\paragraph{Intersezioni con gli assi}
Troviamo dove la funzione \(f\) interseca gli assi \(x\) e \(y\).\\
Vedi sopra; non è fondamentale...\\\\
E indovina un po'? Anche qui lo saltiamo.
\paragraph{Asintoti verticali e orizzontali}
Vediamo se la funzione ha degli asintoti.\\
Troviamo tutti i limiti rilevanti di \(f\).\\
A \(+\infty\) e a \(-\infty\), in punti di non derivabilità, etc...\\\\
\[\lim_{x \to +\infty} (2 arctan(x) - x) = -\infty\]
Essendo una funzione dispari, allora...
\[\lim_{x \to -\infty} f(x) = +\infty\]
\paragraph{Asintoto obliquo}
Controlliamo se esiste un asintoto obliquo.\\
Non è fondamentale, ma potrebbe essere interessante da calcolare.\\
E' presente solo se \(\lim_{x \to \infty} = \pm\infty\).\\
Se lo fa, possiamo calcolarlo.\\\\
\[m = \lim_{x \to +\infty} \frac{2 arctan(x) - x}{x} = -1\]
\[q = \lim_{x \to +\infty} (2 arctan(x) - x) + x = \pi\]
Dunque, l'asintoto obliquo è la retta \(y = -x + \pi\).
\subsubsection{Derivata prima \(f'\)}
\[f'(x) = \frac{2}{1 + x^2} - 1\]
\paragraph{Crescenza}
Troviamo dove la funzione è crescente o decrescente.\\
\[\frac{2 - 1 - x^2}{1 + x^2} \geq 0\]
\[\frac{1 - x^2}{1 + x^2} \geq 0\]
\[x^2 \leq 1\]
\[-1 \leq x \leq 1\]
\paragraph{Punti di estremo}
Troviamo i punti di massimo e i punti di minimo, e se possibile il loro valore.\\\\
Nel nostro caso, \(x = -1\) è un punto di minimo locale e \(x = 1\) è un punto di massimo locale.\\
Vediamo quanto valgono:
\[f(1) = 2 arctan(1) - 1 = 2 \frac{\pi}{4} - 1 = \frac{\pi - 2}{2} \approx 0.6\]
\[f(-1) = 2 arctan(-1) + 1 = - 2 \frac{\pi}{4} + 1 = \frac{- \pi + 2}{2} \approx -0.6\]
\subsubsection{Derivata seconda \(f''\)}
Potrebbe non essere richiesta, se si creerebbe un calcolo complicato.
\[f''(x) = -\frac{4x}{(1 + x^2)^2}\]
\paragraph{Concavità}
Troviamo dove la funzione è concava e dove è convessa.\\
\[-\frac{4x}{(1 + x^2)^2} \geq 0\]
\[x \geq 0\]
\paragraph{Punti di flesso}
Troviamo i punti di flesso:\\\\
Nel nostro caso, l'unico è \(x = 0\).
\subsection{Esercizio}
Fai un grafico qualitativo di \(log | 4 - x | + \frac{2}{|x - 4|}\).
\paragraph{Simmetrie}
E' simmetrica per l'asse \(x = 4\), ma il punto nell'asse stesso è fuori dal dominio.
Possiamo però traslare il tutto ponendo \(x - 4 = t\)...
\[f(t) = \log(|t|) + \frac{2}{|t|}\].
Ora la funzione \(f(t)\) è pari.
\paragraph{Dominio}
\[{t \in \mathbb{R} : t \neq 0}\]
\paragraph{Positività}
\begin{quote}
E' un casino!
\end{quote}
\paragraph{Limiti}
[todo]
\subsubsection{Derivata prima}
\begin{quote}
Il valore assoluto è una specie protetta; gli informatici non hanno la licenza di derivarlo.
\end{quote}
Dividiamo la funzione in casi.
\[\tilde{f}(t) = \begin{cases}
\log t + \frac{2}{t} \qquad t > 0\\
\log (-t) - \frac{2}{t} \qquad t < 0
\end{cases}\]
Deriviamo i due rami separatamente:
\[\tilde{f}'(t) = \begin{cases}
\frac{1}{t} - \frac{2}{t^2} \qquad t > 0\\
[todo] \qquad t < 0
\end{cases}\]
\subsection{Studio di funzione qualitativo in un punto}
Esiste, ma non l'abbiamo fatto.
\end{document}

BIN
public/materials/year1/analisi/8_serie.pdf (Stored with Git LFS) Normal file

Binary file not shown.

View file

@ -0,0 +1,202 @@
\documentclass{article}
\usepackage[utf8]{inputenc}
\usepackage{mathtools}
\usepackage{amssymb}
\usepackage{centernot}
% New symbols
\let\oldsqrt\sqrt
\def\sqrt{\mathpalette\DHLhksqrt}
\def\DHLhksqrt#1#2{%
\setbox0=\hbox{$#1\oldsqrt{#2\,}$}\dimen0=\ht0
\advance\dimen0-0.2\ht0
\setbox2=\hbox{\vrule height\ht0 depth -\dimen0}
{\box0\lower0.4pt\box2}}
% End new symbols
\begin{document}
\section{Le Serie}
\[\sum^{\infty}_{n=0}a_n\]
Se \(\sum^{\infty}_{n=0}a_n = L\), la serie è \textbf{convergente}; se \(\sum^{\infty}_{n=0}a_n = \infty\), la serie è \textbf{divergente}.
\subsection{Condizione necessaria}
\[\sum^{\infty}_{n=0}a_n < +\infty \quad \implies \quad a_n \to 0\]
\[a_n \not\to 0 \quad \implies \quad \sum^{\infty}_{n=0}a_n non convergente\]
\subsection{Serie a termini non negativi definitivamente}
\[\sum^{\infty}_{n=0}a_n \qquad a_n \geq 0\]
Se la successione delle somme parziali è \textit{definitivamente} monotona, allora \textbf{ha limite}, e quindi \textbf{esiste}, convergendo o divergendo.\\
Possiamo applicare dei particolari criteri per capirlo.
\subsection{Criteri}
\subsubsection{Criterio del confronto}
Siano \(\{a_n\}\) e \(\{b_n\}\) due successioni a termini reali \textit{non negativi}, tali che \textit{definitivamente} \(a_n \leq b_n\).\\
Allora...
\[\sum^{\infty}_{n=0} b_n < +\infty \implies \sum a_n < +\infty\]
\[\sum^{\infty}_{n=0} a_n = +\infty \implies \sum b_n = +\infty\]
Si usa principalmente quando la serie converge ma non è dimostrabile convenzionalmente.
\subsubsection{Criterio del confronto asintotico}
Siano \(\{a_n\}\) e \(\{b_n\}\) due successioni a termini reali \textit{positivi}, tali che \(a_n \sim b_n\).\\
Allora \(\sum a_n\) e \(\sum b_n\) hanno lo stesso carattere (entrambe convergono, entrambe divergono, etc).\\
\\
Solitamente si applica per i limiti notevoli.
\subsubsection{Criterio del rapporto}
\[\lim_{n \to +\infty} \frac{a_{n+1}}{a_n} =
\begin{cases}
L < 1 \quad \implies \quad \sum a_n \neq \infty\\
L > 1 \quad \implies \quad \sum a_n = \infty\\
L = 1 \quad \implies \quad unknown
\end{cases}\]
\subsubsection{Criterio della radice}
Sia \(\{a_n\}_{n \in \mathbb{N}^+}\).\\
Supponiamo che \(\lim_{n \to +\infty} \sqrt{a_n}^n = L\).
Allora...
\[\begin{cases}
L < 1 \quad \implies \quad \sum a_n convergente\\
L > 1 \quad \implies \quad \sum a_n divergente\\
L = 1 \quad \quad unknown
\end{cases}\]
\subsection{Serie a termini qualunque}
\subsubsection{Criterio di Leibniz}
\[\sum^{\infty}_{n=0} (-1)^n a_n\]
Se:
\[\begin{cases}
a_n \geq 0
a_n \geq a_{n+1}
a_n \to 0
\end{cases}\]
\subsubsection{Criterio di convergenza assoluta}
Se:
\[\sum^{\infty}_{n=0} |a_n| = \infty\]
Allora:
\[\sum^{\infty}_{n=0} a_n = \infty\]
\subsection{Dimostrazione dei criteri}
\subsubsection{Criterio del confronto}
\[S_n = \sum^n_{k=1} a_k\]
\[S_n^* = \sum^n_{k=1} b_k\]
\subsubsection{Criterio del confronto asintotico}
\[\lim_{n \to +\infty} \frac{a_n}{b_n} = 1\]
Usiamo la definizione di limite:
\[\forall \epsilon > 0 \exists n' : \forall n \geq n', 1 - \epsilon \leq \frac{a_n}{b_n} \leq 1 + \epsilon\]
\[b_n * (1 - \epsilon) \leq \frac{a_n}{b_n} \leq b_n * (1 + \epsilon)\]
Ho ora un'espressione a cui è applicabile il criterio del confronto.
Per la proprietà di monotonia:
\[0 \leq a_k \leq b_k \quad \implies \quad 0 \leq S_n \leq S_n^*\]
\subsubsection{Criterio della radice}
\[\forall \epsilon > 0, \exists n' : \forall n \geq n', L - \frac{\epsilon}{2} \leq \sqrt{a_n}^n \leq L + \frac{\epsilon}{2}\]
Per il funzionamento stesso della radice:
\[L < 1 \implies \exists \epsilon > 0 : L + \epsilon < 1; L < 1 - \epsilon\]
Dunque...
\[\sqrt{a_n}^n \leq 1 - \epsilon + \frac{\epsilon}{2} = 1 - \frac{\epsilon}{2}\]
Ho finalmente raggiunto un punto in cui posso usare il criterio del confronto:
\[\sum a_n \leq \sum (1 - \frac{\epsilon}{2})^2\]
\section{Tipi di esercizi}
Gli esercizi con le serie principalmente sono di tre tipi: calcolare la somma (il valore) di una serie, studiare la convergenza di una serie e studiare la convergenza di una serie che varia in base a un parametro.\\
\subsection{Serie geometriche}
\[\sum^{\infty}_{n=0}q^n\qquad se |q| < 1 \quad = \frac{1}{1-q}\]
\subsubsection{Esempio serie geometrica}
Calcolare \(\sum^{\infty}_{n=0}\frac{1}{2^n}\).\\
\subsection{Serie armonica generalizzata}
\[\sum^{\infty}_{n=0} \frac{1}{n^\alpha} \quad
\begin{cases}
\neq \infty \quad se \quad \alpha > 1\\
= \infty \quad se \quad \alpha \leq 1
\end{cases}
\]
\paragraph{Svolgimento}
E' una serie geometrica di ragione \(\frac{1}{2}\), quindi la somma vale \(\frac{1}{1-\frac{1}{2}} = 2\).
\subsubsection{Serie geometrica nascosta}
Calcolare \(\sum^{\infty}_{n=0}\frac{2^{n-1}}{3^n}\).\\
\paragraph{Svolgimento}
C'è una serie geometrica nascosta: è possibile convertire la somma in \(\frac{1}{2} \sum^{\infty}_{n=0}(\frac{2}{3})^n\), che è una serie geometrica di ragione \(\frac{2}{3}\).\\
Dunque, la somma vale \(\frac{1}{1-\frac{2}{3}} = \frac{3}{2}\).
\subsubsection{Serie geometrica con inizio spostato}
Calcolare \(\sum^{\infty}_{n=1}(\log(3) - 1)^n\).
\paragraph{Svolgimento}
Verifichiamo che la ragione sia effettivamente \(< 1\): \(log(3) - 1 < 1\) è vero.\\
Si converte la serie in \(\sum^{\infty}_{n=0}((\log(3) - 1)^n) - 1\).\\
E' diventata una serie geometrica di ragione \(\log(3) - 1\) a cui dovrà essere sottratto 1 dal risultato finale.
\subsection{Condizione necessaria}
Studia la convergenza di \(\sum^{\infty}_{n=1}(1 + \frac{1}{n!})^n\).
\paragraph{Svolgimento}
\[\sum^{\infty}_{n=1} (e^{n log(1 + \frac{1}{n!})})\]
\[\sum^{\infty}_{n=1} (e^{n \frac{1}{n!})})\]
\[e^{n \frac{1}{n!}} \to 1\]
Dato che l'argomento delle serie non è infinitesimo, allora possiamo dire che la serie non converge.
\subsection{Dipendenti da parametro}
Calcolare per quali valori di x la serie seguente converge.
\[\sum^{\infty}_{n=1} (\frac{x-2}{4})^n\]
\paragraph{Svolgimento}
Riconosciamo che è una serie geometrica, e sappiamo che converge se la sua ragione è \(|r| < 1\).\\
Calcoliamo per quali valori è presente quella ragione:
\[| \frac{x-2}{4} | < 1\]
\[-1 < \frac{x-2}{4} | < 1\]
\[-2 < x < 6\]
\subsection{Criterio del confronto difficile}
\[\sum^{\infty}_{n=1} \frac{\log^2 n}{n \sqrt{n}}\]
\[\lim_{n \to +\infty} \frac{\log^2 n}{n \sqrt{n}} = 0\]
Non concludo nulla da questo limite; devo usare un criterio.
\[\sum^{\infty}_{n=1} \frac{\log^2 n}{n \sqrt{n}}\]
\[\sum^{\infty}_{n=1} \frac{\log^2 n}{n^{\frac{3}{2}}}\]
\[\lim_{n \to \infty} \frac{log n}{n^\alpha} = 0\]
\[\log n \leq n^\alpha\]
\[\log n \leq n^\frac{1}{8}\]
\[\log^2 n \leq n^\frac{1}{4}\]
\[\frac{log^2 n}{n \sqrt{n}} \leq \frac{n^\frac{1}{4}}{n \sqrt{n}} = \frac{1}{n^{\frac{5}{4}}}\]
Applichiamo poi il teorema di confronto.
[TBD]
\subsection{Criterio di confronto asintotico difficile}
Determinare per quali valori di \(\alpha > 0\) la serie converge.
\[\sum^\infty_{n=1} \frac{1 + e^{-n}}{\sqrt{n^\alpha} + log n}\]
Non posso usare la condizione necessaria, perchè \(a_n \to 0\).\\
Applico il criterio del confronto asintotico.
\[a_n \sim \frac{1}{n^\frac{\alpha}{2}}\]
E' una serie armonica generalizzata.\\
Per \(\alpha > 2\), la serie converge, mentre per \(\alpha \leq 2\) la serie diverge.
\subsection{Criterio della radice}
\[\sum_{n=1}^\infty \frac{e^{n^2}}{n^{2n}}\]
\[\sqrt{a_n}^n = (\frac{e^{n^2}}{n^{2n}}^{\frac{1}{n}}\]
\[= \frac{e^n}{n^2} \to +\infty\]
\subsection{Criterio del rapporto}
\[\sum^\infty_{n=1} \frac{e^{n^2}}{n^{2n}}\]
\[\frac{a_{n+1}}{a_n} \to L\]
\[\frac{e^{(n+1)^2}}{(n+1)^{2(n+1)}} * \frac{n^{2n}}{e^{n^2}}\]
\[\frac{e^{2n+1}}{(n+1)^2} * (\frac{n}{n+1})^2n\]
\[\frac{e^{2n+1}}{(n+1)^2} * (\frac{1}{1+\frac{1}{n})^2n}\]
\[\lim_{n \to \infty} \frac{e^{2n+1}}{(n+1)^2} * (\frac{1}{1+\frac{1}{n})^2n}\]
\[+\infty * \frac{1}{e} = +\infty\]
\(+\infty > 1\), dunque la serie diverge.
\end{document}

BIN
public/materials/year1/analisi/9_numeri_complessi.pdf (Stored with Git LFS) Normal file

Binary file not shown.

View file

@ -0,0 +1,94 @@
\documentclass{article}
\usepackage[utf8]{inputenc}
\usepackage{mathtools}
\usepackage{amssymb}
\usepackage{centernot}
% New symbols
\let\oldsqrt\sqrt
\def\sqrt{\mathpalette\DHLhksqrt}
\def\DHLhksqrt#1#2{%
\setbox0=\hbox{$#1\oldsqrt{#2\,}$}\dimen0=\ht0
\advance\dimen0-0.2\ht0
\setbox2=\hbox{\vrule height\ht0 depth -\dimen0}
{\box0\lower0.4pt\box2}}
\newcommand{\iu}{\mathrm{i}\mkern1mu}
% End new symbols
\begin{document}
\section{Equazioni in \(\mathbb{C}\)}
Come possiamo fare a risolvere equazioni in numeri complessi?\\
Una possibile soluzione è quella di applicare la definizione di numero complesso \(z = a + \iu b\).\\
Effettuiamo le seguenti sostituzioni:
\[Re z = a\]
\[Im z = b\]
\[z = a + \iu b\]
\[\bar{z} = a - \iu b\]
Spostando tutti gli elementi al primo membro, giungeremo ad avere al secondo membro \(= 0 + 0i\); possiamo allora fare un sistema con la parte reale e la parte immaginaria del primo membro e risolverlo per a e b; infine, dovremo verificare manualmente tutte le soluzioni trovate in questo modo.
\subsection{Esempio}
[todo, non l'ho copiato]
\subsection{Altro esempio}
\[\begin{cases}
z \bar{w} = \iu\\
|z|^2 w + z = 1
\end{cases}\]
Passiamo la seconda equazione ai coniugati.
\[\bar{|z|^2 w + z = 1} = \bar{1} = 1\]
\[\bar{|z|^2} \bar{w} + \bar{z} = 1\]
Vado a ricavare \(z\) dalla prima equazione.\\
Se \(z \neq 0\), allora...
\[\bar{w} = \frac{\iu}{z}\]
E obbligatoriamente \(z \neq 0\), perchè altrimenti l'equazione non sarebbe verificata.\\
Sappiamo che il modulo \(|z|^2 = (Re z)^2 + (Im z)^2 = z \bar{z}\), dunque tornando alla seconda equazione:
\[\frac{z \bar{z} \iu}{z} + \bar{z} = 1\]
\[\bar{z} \iu + \bar{z} = 1\]
Risolvo la seconda equazione:
\[a\iu + a + b - b\iu - 1 = 0\]
\[\begin{cases}
a + b - 1 = 0\\
a = b
\end{cases}\]
\[a = b = \frac{1}{2}\]
\[z = \frac{1}{2} + \frac{\iu}{2}\]
\[\bar{w} = \frac{\iu}{z} = \frac{\iu}{\frac{1}{2} + \frac{\iu}{2}} = \frac{2 \iu + 2}{2} = 1 + \iu\]
\section{Forma trigonometrica dei numeri complessi}
Possiamo rappresentare i numeri complessi in un'altra forma, invece che quella algebrica.\\
Rappresentiamo un complesso composto da un modulo \(\rho\) e un argomento \(\theta\) corrispondente all'angolo formato da il semiasse positivo del piano cartesiano e la semiretta che congiunge z e l'origine.
\[\rho = \sqrt{a^2 + b^2}\]
\[\begin{cases}
a = Re z = |z| \cos(\theta)\\
b = Im z = |z| \sin(\theta)
\end{cases}\]
[esempi omessi tanto sono sulle dispense]
\section{Teorema}
Siano \(z = \rho (\cos(\theta) + \iu \sin(\theta))\) e \(w = r (\cos \phi + \iu \sin(\theta))\), allora:
\[zw = \rho r (\cos(\theta + \phi) + \iu \sin(\theta + \phi)\]
\[\frac{z}{w} = \frac{\rho}{r} (\cos(\theta - \phi) + \iu \sin(\theta - \phi))\]
\subsection{Potenza di un complesso}
\[z^n = \rho^n (\cos(n \rho) + \iu sin (n \rho))\]
\subsubsection{Esempio}
Calcolare \((1 + \iu)^{16}\).
\paragraph{Svolgimento}
Troviamo la forma trigonometrica:
\[1 + \iu = \sqrt{2} (\cos(\frac{\pi}{4}) + \iu \sin({\pi}{4}))\]
\[(1 + \iu)^{16} = 2^4 (\cos(4 \pi) + \iu sin(4 \pi))\]
\subsubsection{Esempio}
\[i^{2018} = i^{504 * 4} * i^{2} = -1\]
\section{Radici ennesime di numeri complessi}
Sia \(w \in \mathbb{C}, w \neq 0\), allora esistono \(n\) radici ennesime complesse \(z_0, z_1, ..., z_{n-1}\) di \(w\), tali che:
\[z^n_i = w \qquad i=0, ..., n-1\]
Inoltre:
\[w = r (\cos(\phi) + \iu \sin(\phi))\]
\[z_K = \rho_K (\cos(\phi_K) + \iu \sin(\phi_K)) \qquad k = 0, ..., n-1\]
\[\rho_K = r^{\frac{1}{n}}\]
\[\phi_K = \frac{\phi}{n} + \frac{2 \pi K}{n}\]
\end{document}

View file

@ -0,0 +1,28 @@
### Proposizione
Una successione _definitivamente_ limitata e' (sempre) **limitata**.
##### Ipotesi
a(n) definitivamente limitata
Esistono `M` e `p` tale che, per ogni n maggiore di p, a(n) <= M.
`\exists M, p | \forall n \geq p, a(n) \leq M`
##### Tesi
a(n) limitata
Esiste `M'` tale che, per ogni n appartenente ai naturali, a(n) <= M'.
`\exists M' | \forall n \in N, a(n) \leq M`
##### Dimostrazione
Sia A l'insieme dei risultati di a(n) per tutti i numeri naturali minori di p.
A e' non-vuoto; ha un numero finito di elementi: dunque, esiste `max A = M'`.
Se `n >= p`, `a(n) \leq M \leq M'`.
Se `n < p`, `a(n) \leq M'`.
In generale, quindi, `a(n) \leq M`.
### Successione convergente (__fondamentale per l'esame__)
Una successione a(n) e' **convergente** se `\exists l \in R | \forall \epsilon > 0, \exists m : \forall n \geq m, abs(a(n) - l) < \epsilon`, ovvero `l - \epsilon < a(n) < l + \epsilon`

BIN
public/materials/year1/analisi/X_teoremiprincipali.pdf (Stored with Git LFS) Normal file

Binary file not shown.

View file

@ -0,0 +1,288 @@
% !TeX root = ./teoremiprincipali.tex
\documentclass{article}
\usepackage[utf8]{inputenc}
\usepackage{mathtools}
\usepackage{amssymb}
\usepackage{centernot}
\usepackage{bm}
\usepackage{fullpage}
\usepackage{multicol}
\begin{document}
\section{Teorema di Bolzano-Weierstrass}
\begin{multicols}{2}
\subsection{Ipotesi}
Successione \(a_n\) \textbf{limitata} (superiormente e inferiormente).
\columnbreak
\subsection{Tesi}
Esistono \textsc{infinite sottosuccessioni} (relative alla successione iniziale) convergenti.
\end{multicols}
\subsection{Dimostrazione}
Chiamiamo \(A_0\) l'insieme che contiene tutti i punti della successione.\\
Eseguiamo la seguente procedura per \(k = 0\).
\begin{enumerate}
\item Chiamiamo \([\alpha_k, \beta_k]\) i due estremi dell'intervallo della successione.\\
Prendiamo il punto medio tra i due estremi, e chiamiamolo \(\gamma_k\).\\
Osserviamo che \(\alpha_k \leq \gamma_k \leq \beta_k\), e che la dimensione \(d_k\) dell'intervallo \([\alpha_k, \gamma_k] = [\gamma_k, \beta_k]\) è la metà di \([\alpha_k, \beta_k]\).
\item Creiamo due insiemi di punti della successione: uno con i punti tra \([\alpha_k, \gamma_k]\) e uno con i punti tra \(]\gamma_k, \beta_k]\).
\item Almeno uno dei due insiemi ha un numero infinito di punti: prendiamolo, e chiamiamolo \(A_{k+1}\).
\end{enumerate}
Possiamo ripetere questa procedura un numero infinito di volte: possiamo notare che le dimensioni dell'intervallo \(d_k = (\frac{d_0}{2^k}) \to 0\); dato che \(A_k\) contiene infiniti punti, possiamo creare una sottosuccessione che includa solo punti contenuti in \(A_k\).\\
Essa sarà convergente per il teorema dei carabinieri ad un valore \(L\) tale che \(\alpha_0 \leq \dots \leq \alpha_k \leq L \leq \beta_k \leq \dots \leq \beta_0\).
\newpage
\section{Polinomio di Taylor con resto di Peano}
\subsection{Definizioni preliminari}
\[P_{n, x_0}(x) = (\sum^{n}_{m = 0} \frac{f^{(m)}(x_0) * (x - x_0)^m}{m!})\]
\begin{multicols}{2}
\subsection{Ipotesi}
Funzione \(f(x) :\ ]a, b[ \to \mathbb{R}\), \textbf{derivabile} \(n\) volte in \(x_0\) e \(n - 1\) volte in \(]a, b[\).\\
Punto \(x_0 \in\ ]a, b[\).
\columnbreak
\subsection{Tesi}
La funzione \(f(x)\) è \textsc{approssimabile} nel punto \(x_0\) con il polinomio \(P_{n, x_0}(x) + o(x - x_0)^n\) di grado \(n\).
\end{multicols}
\subsection{Dimostrazione}
Notiamo che \(P^{n}_{n, x_0}(x_0) = f^{(n)}(x_0)\).\\
Proviamo a calcolare il seguente limite, che ci sarà utile nel prossimo passaggio:
\[\lim_{x \to x_0} \frac{f(x) - P_{n - 1, x_0}(x)}{(x - x_0)^n} =^{DH} \lim_{x \to x_0} \frac{f^{(1)}(x) - P^{(1)}_{n - 1, x_0}(x)}{n * (x - x_0)^{n - 1}} =^{\infty DH} \lim_{x \to x_0} \frac{f^{(n - 1)}(x) - P^{(n - 1)}_{n - 1, x_0}(x)}{n! * (x - x_0)^{1}} =\]
\[= \lim_{x \to x_0} \frac{f^{(n - 1)}(x) - f^{(n - 1)}(x_0)}{n! * (x - x_0)}\]
Ora siamo pronti a calcolare il limite con \(n\) invece che \(n - 1\):
\[\lim_{x \to x_0} \frac{f(x) - P_{n, x_0}(x)}{(x - x_0)^n}\]
Estraiamo un termine dal polinomio:
\[\lim_{x \to x_0} \frac{f(x) - P_{n - 1, x_0}(x) - \frac{f^{(n)}(x_0) * (x - x_0)^n}{n!}}{(x - x_0)^n}\]
Raccogliamo termini in modo da formare il limite precedente:
\[\lim_{x \to x_0} ( \frac{f(x) - P_{n - 1, x_0}(x)}{(x - x_0)^n} - \frac{\frac{f^{(n)}(x_0) * (x - x_0)^n}{n!}}{(x - x_0)^n} ) \]
Facciamo uscire dal limite le costanti:
\[- \frac{f^{(n)}(x_0)}{n!} + \lim_{x \to x_0} \frac{f(x) - P_{n - 1, x_0}(x)}{(x - x_0)^n}\]
Per il limite precedente:
\[- \frac{f^{(n)}(x_0)}{n!} + \lim_{x \to x_0} \frac{f^{(n - 1)}(x) - f^{(n - 1)}(x_0)}{n! * (x - x_0)}\]
Raccogliamo \(\frac{1}{n!}\):
\[\frac{1}{n!} (- f^{(n)}(x_0) + \lim_{x \to x_0} \frac{f^{(n - 1)}(x) - f^{(n - 1)}(x_0)}{(x - x_0)})\]
Abbiamo ottenuto un rapporto incrementale, il che significa che:
\[\frac{1}{n!} (- f^{(n)}(x_0) + f^{(n)}(x_0)) = 0\]
\newpage
\section{Teorema di esistenza degli zeri}
\begin{multicols}{2}
\subsection{Ipotesi}
Funzione \(f(x) : [a_0, b_0] \to \mathbb{R}\) \textbf{continua}.\\
\(f(a_0) = f(b_0)\).
\columnbreak
\subsection{Tesi}
Esiste \textsc{almeno un punto} in cui \(f(x) = 0\).
\end{multicols}
\subsection{Dimostrazione}
Notiamo che \(f(a_0) * f(b_0) \leq 0\) (ovvero è negativa, cioè hanno due segni diversi).\\
Definiamo la seguente procedura:
\begin{enumerate}
\item Bisezioniamo l'intervallo \([a_n, b_n]\) in \([a_n, z_n]\) e \([z_n, b_n]\).
\item Almeno uno dei due intervalli è tale che \(f(inizio) * f(fine) \leq 0\) (negativo).
\item Prendiamo un intervallo per il quale il prodotto precedente è negativo, e chiamiamolo \([a_{n+1}, b_{n+1}]\).
\end{enumerate}
Ripetendo infinite volte la procedura, partendo dall'intervallo \([a_0, b_0]\), otterremo un intervallo sempre più "verticalmente stretto" \([a_n, b_n]\).\\
Possiamo notare che \(a_0 \leq a_n \leq b_n \leq b_0\), e che entrambe le successioni tendono allo stesso numero \(a_n \to x\) e \(b_n \to x\).\\
Calcoliamo nuovamente \(f(a_n) * f(b_n)\): sappiamo che risulta essere \(\leq 0\), ma possiamo sostituire il limite: \(f(x) * f(x) \leq 0\).\\
Dunque, abbiamo che \(f(x)^2 \leq 0\), e quindi che \(\exists x : f(x) = 0\).
\newpage
\section{Teorema di Weierstrass}
\begin{multicols}{2}
\subsection{Ipotesi}
Funzione \(f(x) : [a, b] \to \mathbb{R}\) \textbf{continua}.
\columnbreak
\subsection{Tesi}
\(f(x)\) assume entro \([a, b]\) un \textsc{valore massimo} e un \textsc{valore minimo}.
\end{multicols}
\subsection{Dimostrazione per il massimo}
Chiamiamo \(M = sup(f)\) l'estremo superiore della funzione f: vogliamo dimostrare che esso è anche il massimo, e che quindi il massimo esiste per la funzione.\\
Dobbiamo quindi \textsc{trovare un valore} \(x\) tale che \(f(x) = M\).\\
Creiamo una successione \(y_n\) che ci aiuti a trovare il valore di \(f(x)\):
\begin{itemize}
\item Se \(M = +\infty\), allora \(y_n = n\) (in modo che la successione \(\to +\infty\)).
\item Se \(M \neq +\infty\), allora \(y_n = M - \frac{1}{n}\) (in modo che la successione \(\to M\)).
\end{itemize}
Possiamo dire che \(y_n < M\), ed essendo \(M\) il minimo dei maggioranti di \(f : [a, b]\):
\[\forall n, \exists x_n : (y_n < f(x_n) \leq M) \land (a < x_n \leq b) \]
Passando al limite, per il \textit{teorema dei carabinieri} abbiamo che \(f(x_n) \to M\).\\
Inoltre, per il \textit{teorema di Bolzano-Weierstrass} sappiamo che esiste una sottosuccessione convergente \(x_{k_n} \to x\) di \(x_n\).\\
Essendo la funzione \textit{continua}, allora \(x_{k_n} \to x \implies f(x_{k_n}) \to f(x)\).\\
Essendo però la sottosuccessione \textit{un'estratta}, allora abbiamo anche che \(f(x_{k_n}) \to M\).\\
Per il \textit{teorema dell'unicità del limite} allora deduciamo che \(M = f(x_{k_n})\), e quindi che \(x_{k_n} = x\).
\subsection{Dimostrazione per il minimo}
La stessa cosa, ma con \(inf(f) = -sup(-f)\).
\newpage
\section{Teorema di Fermat}
\begin{multicols}{2}
\subsection{Ipotesi}
Funzione \(f(x) : [a, b] \to \mathbb{R}\) \textbf{derivabile} in un punto \(x_0 \in\ ]a, b[\).\\
\(x_0\) punto di estremo locale.
\columnbreak
\subsection{Tesi}
\(f'(x_0) = 0\).
\end{multicols}
\subsection{Dimostrazione per il minimo locale}
Sappiamo che se \(x_0\) è un \textbf{minimo locale}, esiste obbligatoriamente un intorno \(I \subset [a, b]\) in cui \(\forall x \in I, f(x_0) \leq f(x)\).\\
Possiamo provare a calcolare il suo rapporto incrementale: \(\lim_{x \to x_0} \frac{f(x) - f(x_0)}{x - x_0}\).\\
Notiamo che mentre il numeratore è sempre positivo, il denominatore cambia in base a se \(x > x_0\).\\
Allora, \(f'_-(x_0) \leq 0\), e \(f'_+(x_0) \geq 0\).\\
Essendo la funzione \textbf{derivabile}, e quindi \(f'_-(x) = f'_+(x)\) l'unica possibilità è che \(f(x_0) = 0\).
\subsection{Dimostrazione per il massimo locale}
La stessa cosa, ma con \(-f\).
\newpage
\section{Teorema di Rolle}
\begin{multicols}{2}
\subsection{Ipotesi}
Funzione \(f(x)\) tale che
\begin{itemize}
\item sia \textbf{continua} in \([a, b]\)
\item sia \textbf{derivabile} in \([a, b]\)
\item \(f(a) = f(b)\)
\end{itemize}
\columnbreak
\subsection{Tesi}
\(\exists x_0 : f'(x_0) = 0\) (ovvero la funzione è \textsc{costante} o ha \textsc{almeno un punto stazionario})
\end{multicols}
\subsection{Dimostrazione}
Se la funzione è \textbf{continua}, allora per il \textit{teorema di Weierstrass} sappiamo che ha almeno un punto di massimo \(x_M\) e uno di minimo \(x_m\) in \([a, b]\).\\
Se i valori di entrambi i due punti coincidono con \(f(a) = f(b)\), allora la funzione è \textsc{costante}.\\
Se almeno uno dei due valori è diverso da \(f(a) = f(b)\), allora per il \textit{teorema di Fermat} \(f'(x_0) = 0\).
\newpage
\section{Teorema di Cauchy}
\begin{multicols}{2}
\subsection{Ipotesi}
Funzioni \(f(x)\) e \(g(x)\) tale che
\begin{itemize}
\item siano \textbf{continue} in \([a, b]\)
\item siano \textbf{derivabili} in \([a, b]\)
\end{itemize}
\columnbreak
\subsection{Tesi}
\(\exists c : ((f(a) - f(b))g'(c) = (g(a) - g(b))f'(c))\)
\end{multicols}
\subsection{Dimostrazione}
Creiamo una funzione \(w\) tale che \(w(x) = (f(a) - f(b))g(x) - (g(a) - g(b))f(x))\).\\
Essendo formata dalla differenza di due funzioni \textbf{continue}, è anche essa continua.\\
Essendo formata dalla differenza di due funzioni \textbf{derivabili}, è anche essa derivabile.\\
Sostituendo, notiamo che \(w(a) = w(b)\).\\
Allora, per il teorema di Rolle, sappiamo che ha un punto stazionario \(c\) tale che \(w'(c) = 0\).\\
Con \(w'(c) = 0\), abbiamo che \(\exists c : ((f(a) - f(b))g'(c) = (g(a) - g(b))f'(c))\).
\subsection{Significato geometrico}
Il significato geometrico del teorema di Cauchy è che presa una qualsiasi curva, essa ha almeno un punto in cui la pendenza è uguale alla pendenza della retta tra i punti a e b.
\newpage
\section{Teorema di Lagrange}
\begin{multicols}{2}
\subsection{Ipotesi}
Funzione \(f(x)\) tale che
\begin{itemize}
\item sia \textbf{continua} in \([a, b]\)
\item sia \textbf{derivabile} in \([a, b]\)
\end{itemize}
\columnbreak
\subsection{Tesi}
\(\exists c : f'(c) = \frac{f(b) - f(a)}{b - a}\)
\end{multicols}
\subsection{Dimostrazione}
Il \textit{Teorema di Cauchy}, con \(g(x) = x\).
\newpage
\section{Teorema della media integrale}
\begin{multicols}{2}
\subsection{Ipotesi}
\begin{enumerate}
\item Funzione \(f(x)\) \textbf{integrabile} in \([a, b]\)
\item Funzione \(f(x)\) \textbf{continua}
\end{enumerate}
\columnbreak
\subsection{Tesi}
\begin{enumerate}
\item \(inf(f) \leq \frac{1}{b - a} \int_a^b f(x) \leq sup(f) \)
\item \(\exists z : (\frac{1}{b - a} \int_a^b f(x) = f(z))\)
\end{enumerate}
\end{multicols}
\subsection{Dimostrazione}
Per la definizione di integrale, \(inf(f) < f(x) < sup(f)\), quindi anche \(inf(f) < \frac{1}{b - a} \int_a^b f(x) < sup(f) \).\\
Se la funzione è anche \textbf{continua}, allora per \textit{Weierstrass} esistono un massimo \(M\) e un minimo \(m\).\\
Allora, \(\forall x, m \leq f(x) \leq M\).\\
Ma per la definizione di integrale, \(m = \int_a^b m dx \leq \int_a^b f(x) dx \leq \int_a^b M dx = M \).\\
E in particolare, \(m \leq \frac{1}{b - a} \int_a^b f(x) dx \leq M\).
\newpage
\section{Teorema fondamentale del calcolo integrale}
\begin{multicols}{2}
\subsection{Ipotesi}
Funzione \(f(x)\) \textbf{integrabile} in \(]a, b[\)\\
Funzione \(G(x) : ]a, b[\) \textbf{primitiva} di \(f(x)\)
\columnbreak
\subsection{Tesi}
\(\int_a^b f(x) dx = G(b) - G(a) = [G(x)]^b_a\)
\end{multicols}
\subsection{Dimostrazione}
Prolunghiamo la primitiva \(G(x)\) per continuità:
\begin{itemize}
\item \(G(a^+) = \lim_{x \to a^+} f(x)\)
\item \(G(b^-) = \lim_{x \to b^-} f(x)\)
\end{itemize}
La primitiva ora è continua in \([a, b]\).\\
Possiamo allora partizionarla in un numero infinito di intervalli \([a, x_i] = \dots = [x_j, b]\).\\
Per il \textit{teorema di Lagrange}, \(\forall\ partizione\ "n" [c, d], \exists z : G(d_n) - G(c_n) = G'(z_n)(d_n - c_n) = f(z_n)(d_n - c_n)\).\\
Allora, possiamo dire che \(G(b) - G(a) = \sum_{j = 0}^n f(z_j)(d_j - c_j) = S_j\).\\
Abbiamo dunque una somma di Cauchy-Riemann, e possiamo dire che \(G(b) - G(a) = \int_a^b f(x)\).
\newpage
\section{Secondo teorema fondamentale del calcolo integrale}
\begin{multicols}{2}
\subsection{Ipotesi}
Funzione \(f(x)\) \textbf{integrabile}.\\
Funzione \(F(x) = \int_{x_0}^x f(x) dx\)
\columnbreak
\subsection{Tesi}
Funzione \(F(x)\) \textsc{continua}
\(f(x)\ continua \implies F'(x) = f(x)\)
\end{multicols}
\end{document}

BIN
public/materials/year1/analisi/X_weierstrass.pdf (Stored with Git LFS) Normal file

Binary file not shown.

View file

@ -0,0 +1,49 @@
\documentclass{article}
\usepackage[utf8]{inputenc}
\usepackage{mathtools}
\usepackage{amssymb}
\usepackage{centernot}
% New symbols
\let\oldsqrt\sqrt
\def\sqrt{\mathpalette\DHLhksqrt}
\def\DHLhksqrt#1#2{%
\setbox0=\hbox{$#1\oldsqrt{#2\,}$}\dimen0=\ht0
\advance\dimen0-0.2\ht0
\setbox2=\hbox{\vrule height\ht0 depth -\dimen0}
{\box0\lower0.4pt\box2}}
\newcommand{\iu}{\mathrm{i}\mkern1mu}
% End new symbols
\begin{document}
\section{Teorema di Weierstrass}
\paragraph{Ipotesi}
\footnotesize Devo scrivere per forza qualcosa qua.\\\normalsize
\([a, b]\) intervallo \textit{chiuso} e \textit{limitato}\\
\(f\) continua su \([a, b]\)
\paragraph{Tesi}
\(f\) ha \textit{massimo} e \textit{minimo} su \([ a, b ]\)\\
\[\exists x_M, \exists x_n : f(x_M) \leq f(x) \leq f(x_n)\]
\paragraph{Tabella lettere}
\footnotesize Per capirci qualcosa in più.\\\normalsize
\(f\) \quad funzione\\
\(M\) \quad reale, estremo superiore della funzione\\
\(x_M\) \quad reale, punto in cui la funzione raggiunge il valore di \(M\)\\
\(x_n\) \quad successione, ???\\
\(y_n\) \quad successione, ???
\paragraph{Dimostrazione}
\footnotesize Dimostrazione per il minimo omessa, in quanto opposta di questa.\\\\\normalsize
Sia \(M = sup(f) = sup \{f(x) : x \in [a, b]\}\).\\\\
Devo dimostrare che M venga raggiunto in almeno un punto della funzione: \(\exists x_M \in [a, b]\) tale che \(f(x_M) = M\).\\\\
\(M\) è il minimo dei maggioranti; se considero un qualsiasi numero \(y_n < M\), questo \textit{non è un maggiorante} per la definizione di estremo superiore.\\\\
Allora, creo una successione \(x_n\) in modo che \(y_n < f(x_n) \leq M\).\\\\
Dato che \(y_n\) tende ad \(M\), per il \textsc{Teorema dei Carabinieri} \(f(x_n) \to M\).\\\\
Il fatto che \(x_n\) sia \(\in [a, b]\) ci fa dire che la successione sia \textit{limitata}.\\\\
Essendo limitata, per il \textsc{Teorema di Bolzano-Weierstrass} possiamo estrarre sicuramente una sottosuccessione \(x_{n_k}\) tale che essa tenda a un valore finito \(\to x_M\).\\\\
Essendo \(f\) una funzione continua, allora \(f(x_{n_k} \to f(x_n)\).\\\\
Dato che tutte le sottosuccessioni estratte tendono allo stesso valore, allora possiamo dire che \(M = f(x_M)\).
\end{document}

4
public/vercel.svg Normal file
View file

@ -0,0 +1,4 @@
<svg width="283" height="64" viewBox="0 0 283 64" fill="none"
xmlns="http://www.w3.org/2000/svg">
<path d="M141.04 16c-11.04 0-19 7.2-19 18s8.96 18 20 18c6.67 0 12.55-2.64 16.19-7.09l-7.65-4.42c-2.02 2.21-5.09 3.5-8.54 3.5-4.79 0-8.86-2.5-10.37-6.5h28.02c.22-1.12.35-2.28.35-3.5 0-10.79-7.96-17.99-19-17.99zm-9.46 14.5c1.25-3.99 4.67-6.5 9.45-6.5 4.79 0 8.21 2.51 9.45 6.5h-18.9zM248.72 16c-11.04 0-19 7.2-19 18s8.96 18 20 18c6.67 0 12.55-2.64 16.19-7.09l-7.65-4.42c-2.02 2.21-5.09 3.5-8.54 3.5-4.79 0-8.86-2.5-10.37-6.5h28.02c.22-1.12.35-2.28.35-3.5 0-10.79-7.96-17.99-19-17.99zm-9.45 14.5c1.25-3.99 4.67-6.5 9.45-6.5 4.79 0 8.21 2.51 9.45 6.5h-18.9zM200.24 34c0 6 3.92 10 10 10 4.12 0 7.21-1.87 8.8-4.92l7.68 4.43c-3.18 5.3-9.14 8.49-16.48 8.49-11.05 0-19-7.2-19-18s7.96-18 19-18c7.34 0 13.29 3.19 16.48 8.49l-7.68 4.43c-1.59-3.05-4.68-4.92-8.8-4.92-6.07 0-10 4-10 10zm82.48-29v46h-9V5h9zM36.95 0L73.9 64H0L36.95 0zm92.38 5l-27.71 48L73.91 5H84.3l17.32 30 17.32-30h10.39zm58.91 12v9.69c-1-.29-2.06-.49-3.2-.49-5.81 0-10 4-10 10V51h-9V17h9v9.2c0-5.08 5.91-9.2 13.2-9.2z" fill="#000"/>
</svg>

After

Width:  |  Height:  |  Size: 1.1 KiB

7
styles/global.css Normal file
View file

@ -0,0 +1,7 @@
body {
margin: 0;
}
#root {
min-height: 100vh;
}

20
tsconfig.json Normal file
View file

@ -0,0 +1,20 @@
{
"compilerOptions": {
"target": "es5",
"lib": ["dom", "dom.iterable", "esnext"],
"allowJs": true,
"skipLibCheck": true,
"strict": true,
"forceConsistentCasingInFileNames": true,
"noEmit": true,
"esModuleInterop": true,
"module": "esnext",
"moduleResolution": "node",
"resolveJsonModule": true,
"isolatedModules": true,
"jsx": "preserve",
"incremental": true
},
"include": ["next-env.d.ts", "**/*.ts", "**/*.tsx"],
"exclude": ["node_modules"]
}

1714
yarn.lock Normal file

File diff suppressed because it is too large Load diff